Patience, young “Padovan”












26












$begingroup$


Everyone knows the Fibonacci sequence:

You take a square, attach an equal square to it, then repeatedly attach a square whose side length is equal to the largest side length of the resulting rectangle.

The result is a beautiful spiral of squares whose sequence of numbers is the Fibonacci sequence:





But, what if we didn't want to use squares?



If we use equilateral triangles—instead of squares—in a similar fashion, we get an equally beautiful spiral of triangles and a new sequence: the Padovan sequence, aka A000931:





Task:



Given a positive integer, $N$, output $a_N$, the $N$th term in the Padovan sequence OR the first $N$ terms.



Assume that the first three terms of the sequence are all $1$. Thus, the sequence will start as follows:
$$
1,1,1,2,2,3,...
$$



Input:




  • Any positive integer $Nge0$


  • Invalid input does not have to be taken into account



Output:




  • The $N$th term in the Padovan sequence OR the first $N$ terms of the Padovan sequence.


  • If the first $N$ terms are printed out, the output can be whatever is convenient (list/array, multi-line string, etc.)


  • Can be either $0$-indexed or $1$-indexed



Test Cases:

(0-indexed, $N$th term)



Input | Output
--------------
0 | 1
1 | 1
2 | 1
4 | 2
6 | 4
14 | 37
20 | 200
33 | 7739


(1-indexed, first $N$ terms)



Input | Output
--------------
1 | 1
3 | 1,1,1
4 | 1,1,1,2
7 | 1,1,1,2,2,3,4
10 | 1,1,1,2,2,3,4,5,7,9
12 | 1,1,1,2,2,3,4,5,7,9,12,16


Rules:




  • This is code-golf: the fewer bytes, the better!


  • Standard loopholes are forbidden.











share|improve this question











$endgroup$








  • 2




    $begingroup$
    14 (0-indexed) is shown as outputting 28 while I believe it should yield 37
    $endgroup$
    – Jonathan Allan
    yesterday










  • $begingroup$
    @JonathanAllan yes, you are correct. I fixed the last two test cases for $N$th term but not that one. The post has been edited.
    $endgroup$
    – Tau
    yesterday










  • $begingroup$
    @LuisMendo I believe so. I'll edit the post.
    $endgroup$
    – Tau
    16 hours ago










  • $begingroup$
    Not to detract from the question, but is this the actual definition of the Fibonacci sequence? I was taught it as a sequence of numbers, in which the first two numbers are 1, and the 3rd and subsequent numbers are the sum of the prior two numbers. Then again, I was taught this as an example of a problem to solve with recursion...
    $endgroup$
    – sharur
    5 hours ago






  • 1




    $begingroup$
    @sharur this definition for the Fibonacci sequence is the visual definition. Each successive square added has a length of that term in the sequence. The sequence you describe is the numerical reasoning behind it. Both sequences work just as well as the other.
    $endgroup$
    – Tau
    3 hours ago
















26












$begingroup$


Everyone knows the Fibonacci sequence:

You take a square, attach an equal square to it, then repeatedly attach a square whose side length is equal to the largest side length of the resulting rectangle.

The result is a beautiful spiral of squares whose sequence of numbers is the Fibonacci sequence:





But, what if we didn't want to use squares?



If we use equilateral triangles—instead of squares—in a similar fashion, we get an equally beautiful spiral of triangles and a new sequence: the Padovan sequence, aka A000931:





Task:



Given a positive integer, $N$, output $a_N$, the $N$th term in the Padovan sequence OR the first $N$ terms.



Assume that the first three terms of the sequence are all $1$. Thus, the sequence will start as follows:
$$
1,1,1,2,2,3,...
$$



Input:




  • Any positive integer $Nge0$


  • Invalid input does not have to be taken into account



Output:




  • The $N$th term in the Padovan sequence OR the first $N$ terms of the Padovan sequence.


  • If the first $N$ terms are printed out, the output can be whatever is convenient (list/array, multi-line string, etc.)


  • Can be either $0$-indexed or $1$-indexed



Test Cases:

(0-indexed, $N$th term)



Input | Output
--------------
0 | 1
1 | 1
2 | 1
4 | 2
6 | 4
14 | 37
20 | 200
33 | 7739


(1-indexed, first $N$ terms)



Input | Output
--------------
1 | 1
3 | 1,1,1
4 | 1,1,1,2
7 | 1,1,1,2,2,3,4
10 | 1,1,1,2,2,3,4,5,7,9
12 | 1,1,1,2,2,3,4,5,7,9,12,16


Rules:




  • This is code-golf: the fewer bytes, the better!


  • Standard loopholes are forbidden.











share|improve this question











$endgroup$








  • 2




    $begingroup$
    14 (0-indexed) is shown as outputting 28 while I believe it should yield 37
    $endgroup$
    – Jonathan Allan
    yesterday










  • $begingroup$
    @JonathanAllan yes, you are correct. I fixed the last two test cases for $N$th term but not that one. The post has been edited.
    $endgroup$
    – Tau
    yesterday










  • $begingroup$
    @LuisMendo I believe so. I'll edit the post.
    $endgroup$
    – Tau
    16 hours ago










  • $begingroup$
    Not to detract from the question, but is this the actual definition of the Fibonacci sequence? I was taught it as a sequence of numbers, in which the first two numbers are 1, and the 3rd and subsequent numbers are the sum of the prior two numbers. Then again, I was taught this as an example of a problem to solve with recursion...
    $endgroup$
    – sharur
    5 hours ago






  • 1




    $begingroup$
    @sharur this definition for the Fibonacci sequence is the visual definition. Each successive square added has a length of that term in the sequence. The sequence you describe is the numerical reasoning behind it. Both sequences work just as well as the other.
    $endgroup$
    – Tau
    3 hours ago














26












26








26


2



$begingroup$


Everyone knows the Fibonacci sequence:

You take a square, attach an equal square to it, then repeatedly attach a square whose side length is equal to the largest side length of the resulting rectangle.

The result is a beautiful spiral of squares whose sequence of numbers is the Fibonacci sequence:





But, what if we didn't want to use squares?



If we use equilateral triangles—instead of squares—in a similar fashion, we get an equally beautiful spiral of triangles and a new sequence: the Padovan sequence, aka A000931:





Task:



Given a positive integer, $N$, output $a_N$, the $N$th term in the Padovan sequence OR the first $N$ terms.



Assume that the first three terms of the sequence are all $1$. Thus, the sequence will start as follows:
$$
1,1,1,2,2,3,...
$$



Input:




  • Any positive integer $Nge0$


  • Invalid input does not have to be taken into account



Output:




  • The $N$th term in the Padovan sequence OR the first $N$ terms of the Padovan sequence.


  • If the first $N$ terms are printed out, the output can be whatever is convenient (list/array, multi-line string, etc.)


  • Can be either $0$-indexed or $1$-indexed



Test Cases:

(0-indexed, $N$th term)



Input | Output
--------------
0 | 1
1 | 1
2 | 1
4 | 2
6 | 4
14 | 37
20 | 200
33 | 7739


(1-indexed, first $N$ terms)



Input | Output
--------------
1 | 1
3 | 1,1,1
4 | 1,1,1,2
7 | 1,1,1,2,2,3,4
10 | 1,1,1,2,2,3,4,5,7,9
12 | 1,1,1,2,2,3,4,5,7,9,12,16


Rules:




  • This is code-golf: the fewer bytes, the better!


  • Standard loopholes are forbidden.











share|improve this question











$endgroup$




Everyone knows the Fibonacci sequence:

You take a square, attach an equal square to it, then repeatedly attach a square whose side length is equal to the largest side length of the resulting rectangle.

The result is a beautiful spiral of squares whose sequence of numbers is the Fibonacci sequence:





But, what if we didn't want to use squares?



If we use equilateral triangles—instead of squares—in a similar fashion, we get an equally beautiful spiral of triangles and a new sequence: the Padovan sequence, aka A000931:





Task:



Given a positive integer, $N$, output $a_N$, the $N$th term in the Padovan sequence OR the first $N$ terms.



Assume that the first three terms of the sequence are all $1$. Thus, the sequence will start as follows:
$$
1,1,1,2,2,3,...
$$



Input:




  • Any positive integer $Nge0$


  • Invalid input does not have to be taken into account



Output:




  • The $N$th term in the Padovan sequence OR the first $N$ terms of the Padovan sequence.


  • If the first $N$ terms are printed out, the output can be whatever is convenient (list/array, multi-line string, etc.)


  • Can be either $0$-indexed or $1$-indexed



Test Cases:

(0-indexed, $N$th term)



Input | Output
--------------
0 | 1
1 | 1
2 | 1
4 | 2
6 | 4
14 | 37
20 | 200
33 | 7739


(1-indexed, first $N$ terms)



Input | Output
--------------
1 | 1
3 | 1,1,1
4 | 1,1,1,2
7 | 1,1,1,2,2,3,4
10 | 1,1,1,2,2,3,4,5,7,9
12 | 1,1,1,2,2,3,4,5,7,9,12,16


Rules:




  • This is code-golf: the fewer bytes, the better!


  • Standard loopholes are forbidden.








code-golf number sequence






share|improve this question















share|improve this question













share|improve this question




share|improve this question








edited 16 hours ago







Tau

















asked yesterday









TauTau

911515




911515








  • 2




    $begingroup$
    14 (0-indexed) is shown as outputting 28 while I believe it should yield 37
    $endgroup$
    – Jonathan Allan
    yesterday










  • $begingroup$
    @JonathanAllan yes, you are correct. I fixed the last two test cases for $N$th term but not that one. The post has been edited.
    $endgroup$
    – Tau
    yesterday










  • $begingroup$
    @LuisMendo I believe so. I'll edit the post.
    $endgroup$
    – Tau
    16 hours ago










  • $begingroup$
    Not to detract from the question, but is this the actual definition of the Fibonacci sequence? I was taught it as a sequence of numbers, in which the first two numbers are 1, and the 3rd and subsequent numbers are the sum of the prior two numbers. Then again, I was taught this as an example of a problem to solve with recursion...
    $endgroup$
    – sharur
    5 hours ago






  • 1




    $begingroup$
    @sharur this definition for the Fibonacci sequence is the visual definition. Each successive square added has a length of that term in the sequence. The sequence you describe is the numerical reasoning behind it. Both sequences work just as well as the other.
    $endgroup$
    – Tau
    3 hours ago














  • 2




    $begingroup$
    14 (0-indexed) is shown as outputting 28 while I believe it should yield 37
    $endgroup$
    – Jonathan Allan
    yesterday










  • $begingroup$
    @JonathanAllan yes, you are correct. I fixed the last two test cases for $N$th term but not that one. The post has been edited.
    $endgroup$
    – Tau
    yesterday










  • $begingroup$
    @LuisMendo I believe so. I'll edit the post.
    $endgroup$
    – Tau
    16 hours ago










  • $begingroup$
    Not to detract from the question, but is this the actual definition of the Fibonacci sequence? I was taught it as a sequence of numbers, in which the first two numbers are 1, and the 3rd and subsequent numbers are the sum of the prior two numbers. Then again, I was taught this as an example of a problem to solve with recursion...
    $endgroup$
    – sharur
    5 hours ago






  • 1




    $begingroup$
    @sharur this definition for the Fibonacci sequence is the visual definition. Each successive square added has a length of that term in the sequence. The sequence you describe is the numerical reasoning behind it. Both sequences work just as well as the other.
    $endgroup$
    – Tau
    3 hours ago








2




2




$begingroup$
14 (0-indexed) is shown as outputting 28 while I believe it should yield 37
$endgroup$
– Jonathan Allan
yesterday




$begingroup$
14 (0-indexed) is shown as outputting 28 while I believe it should yield 37
$endgroup$
– Jonathan Allan
yesterday












$begingroup$
@JonathanAllan yes, you are correct. I fixed the last two test cases for $N$th term but not that one. The post has been edited.
$endgroup$
– Tau
yesterday




$begingroup$
@JonathanAllan yes, you are correct. I fixed the last two test cases for $N$th term but not that one. The post has been edited.
$endgroup$
– Tau
yesterday












$begingroup$
@LuisMendo I believe so. I'll edit the post.
$endgroup$
– Tau
16 hours ago




$begingroup$
@LuisMendo I believe so. I'll edit the post.
$endgroup$
– Tau
16 hours ago












$begingroup$
Not to detract from the question, but is this the actual definition of the Fibonacci sequence? I was taught it as a sequence of numbers, in which the first two numbers are 1, and the 3rd and subsequent numbers are the sum of the prior two numbers. Then again, I was taught this as an example of a problem to solve with recursion...
$endgroup$
– sharur
5 hours ago




$begingroup$
Not to detract from the question, but is this the actual definition of the Fibonacci sequence? I was taught it as a sequence of numbers, in which the first two numbers are 1, and the 3rd and subsequent numbers are the sum of the prior two numbers. Then again, I was taught this as an example of a problem to solve with recursion...
$endgroup$
– sharur
5 hours ago




1




1




$begingroup$
@sharur this definition for the Fibonacci sequence is the visual definition. Each successive square added has a length of that term in the sequence. The sequence you describe is the numerical reasoning behind it. Both sequences work just as well as the other.
$endgroup$
– Tau
3 hours ago




$begingroup$
@sharur this definition for the Fibonacci sequence is the visual definition. Each successive square added has a length of that term in the sequence. The sequence you describe is the numerical reasoning behind it. Both sequences work just as well as the other.
$endgroup$
– Tau
3 hours ago










32 Answers
32






active

oldest

votes













1 2
next












36












$begingroup$


Jelly, 10 bytes



9s3’Ẓæ*³FṀ


Try it online!



1-indexed. Computes the largest element of: $$begin{bmatrix}0&0&1 \ 1&0&1 \ 0&1&0end{bmatrix}^n$$
where the binary matrix is conveniently computed as: $$begin{bmatrix}mathsf{isprime}(0)&mathsf{isprime}(1)&mathsf{isprime}(2) \ mathsf{isprime}(3)&mathsf{isprime}(4)&mathsf{isprime}(5) \ mathsf{isprime}(6)&mathsf{isprime}(7)&mathsf{isprime}(8)end{bmatrix}$$



(this is a total coincidence.)



9s3         [[1,2,3],[4,5,6],[7,8,9]]    9 split 3
’ [[0,1,2],[3,4,5],[6,7,8]] decrease
Ẓ [[0,0,1],[1,0,1],[0,1,0]] isprime
æ*³ [[0,0,1],[1,0,1],[0,1,0]]^n matrix power by input
FṀ flatten, maximum





share|improve this answer









$endgroup$









  • 17




    $begingroup$
    this is clearly some kind of voodoo
    $endgroup$
    – Pureferret
    18 hours ago






  • 6




    $begingroup$
    This should be published.
    $endgroup$
    – YSC
    17 hours ago






  • 2




    $begingroup$
    @YSC It has already been published in A000931. I'd never have guess the primes trick:)
    $endgroup$
    – flawr
    14 hours ago










  • $begingroup$
    @flawr ho... I missed it
    $endgroup$
    – YSC
    13 hours ago






  • 1




    $begingroup$
    ...make that "unless someone can golf two bytes off this one" :) (now that I have a 9 byter)
    $endgroup$
    – Jonathan Allan
    12 hours ago





















19












$begingroup$


Oasis, 5 bytes



nth term 0-indexed



cd+1V


Try it online!



Explanation



   1V   # a(0) = 1
# a(1) = 1
# a(2) = 1
# a(n) =
c # a(n-2)
+ # +
d # a(n-3)





share|improve this answer









$endgroup$





















    16












    $begingroup$


    Haskell, 26 bytes





    (l!!)
    l=1:1:1:2:scanl(+)2l


    Try it online! Outputs the n'th term zero-indexed.



    I thought that the "obvious" recursive solution below would be unbeatable, but then I found this. It's similar to the classic golfy expression l=1:scanl(+)1l for the infinite Fibonacci list, but here the difference between adjacent elements is the term 4 positions back. We can more directly write l=1:1:zipWith(+)l(0:l), but that's longer.



    If this challenge allowed infinite list output, we could cut the first line and have 20 bytes.



    27 bytes





    f n|n<3=1|1>0=f(n-2)+f(n-3)


    Try it online!






    share|improve this answer









    $endgroup$





















      14












      $begingroup$


      Jelly,  10 9  8 bytes



      ŻṚm2Jc$S


      A monadic Link accepting n (0-indexed) which yields P(n).



      Try it online!



      How?



      Implements $P(n) = sum_{i=0}^{lfloorfrac{n}2rfloor}binom{i+1}{n-2i}$



      ŻṚm2Jc$S - Link: integer, n       e.g. 20
      Ż - zero range [0, 1, 2, 3, 4, ..., 19, 20]
      Ṛ - reverse [20, 19, ..., 4, 3, 2, 1, 0]
      m2 - modulo-slice with 2 [20, 18, 16, 14, 12, 10, 8, 6, 4, 2, 0] <- n-2i
      $ - last two links as a monad:
      J - range of length [ 1, 2, 3, 4, 5, 6, 7, 8, 9, 10, 11] <- i+1
      c - left-choose-right [ 0, 0, 0, 0, 0, 0, 0, 28,126, 45, 1]
      S - sum 200




      And here is a "twofer"

      ...a totally different method also for 8 bytes (this one is 1-indexed, but much slower):



      3ḊṗRẎ§ċ‘ - Link: n
      3Ḋ - 3 dequeued = [2,3]
      R - range = [1,2,3,...,n]
      ṗ - Cartesian power [[[2],[3]],[[2,2],[2,3],[3,2],[3,3]],[[2,2,2],...],...]
      Ẏ - tighten [[2],[3],[2,2],[2,3],[3,2],[3,3],[2,2,2],...]
      § - sums [ 2, 3, 4, 5, 5, 6, 6,...]
      ‘ - increment n+1
      ċ - count occurrences P(n)





      share|improve this answer











      $endgroup$





















        7












        $begingroup$


        Python 2, 30 bytes





        f=lambda n:n<3or f(n-2)+f(n-3)


        Try it online!



        Returns the n'th term zero indexed. Outputs True for 1.






        share|improve this answer











        $endgroup$





















          5












          $begingroup$


          Wolfram Language (Mathematica), 33 bytes



          a@0=a@1=a@2=1;a@n_:=a[n-2]+a[n-3]   


          1-indexed, returns the nth term



          Try it online!






          share|improve this answer









          $endgroup$





















            5












            $begingroup$


            J, 23 bytes



            -1 byte thanks to ngn and Galen



            closed form, 26 bytes



            0.5<.@+1.04535%~1.32472^<:


            Try it online!



            iterative, 23 bytes



            (],1#._2 _3{ ::1:])^:[#


            Try it online!






            share|improve this answer











            $endgroup$









            • 1




              $begingroup$
              Another 24-byte solution (boring) : (1#.2 3$:@-~])`1:@.(3&>) Try it online!
              $endgroup$
              – Galen Ivanov
              9 hours ago












            • $begingroup$
              23 bytes thanks to ngn 1: -> # : Try it online!
              $endgroup$
              – Galen Ivanov
              9 hours ago












            • $begingroup$
              @GalenIvanov tyvm, that's a great trick.
              $endgroup$
              – Jonah
              8 hours ago



















            4












            $begingroup$


            Python 2, 56 48 bytes





            f=lambda n,a=1,b=1,c=1:n>2and f(n-1,b,c,a+b)or c


            Try it online!



            Returns nth value, 0-indexed.






            share|improve this answer









            $endgroup$





















              4












              $begingroup$


              Retina, 47 42 bytes



              K`0¶1¶0
              "$+"+`.+¶(.+)¶.+$
              $&¶$.(*_$1*
              6,G`


              Try it online! Outputs the first n terms on separate lines. Explanation:



              K`0¶1¶0


              Replace the input with the terms for -2, -1 and 0.



              "$+"+`.+¶(.+)¶.+$
              $&¶$.(*_$1*


              Generate the next n terms using the recurrence relation. *_ here is short for $&*_ which converts the (first) number in the match to unary, while $1* is short for $1*_ which converts the middle number to unary. The $.( returns the decimal sum of its unary arguments, i.e. the sum of the first and middle numbers.



              6,G`


              Discard the first six characters, i.e. the first three lines.






              share|improve this answer











              $endgroup$





















                4












                $begingroup$


                Cubix, 20 bytes



                This is 0 indexed and outputs the Nth term



                ;@UOI010+p?/sqq;W.(


                Try it online!



                Wraps onto a cube with side length 2



                    ; @
                U O
                I 0 1 0 + p ? /
                s q q ; W . (
                . .
                . .


                Watch it run





                • I010 - Initiates the stack


                • +p? - Adds the top of stack, pulls the counter from the bottom of stack and tests


                • /;UO@ - If counter is 0, reflect onto top face, remove TOS, u-turn, output and halt


                • (sqq;W - If counter is positive, reflect, decrement counter, swap TOS, push top to bottom twice, remove TOS and shift lane back into the main loop.






                share|improve this answer









                $endgroup$





















                  4












                  $begingroup$


                  Octave / MATLAB, 35 33 bytes





                  @(n)[1 filter(1,'cbaa'-98,2:n<5)]


                  Outputs the first n terms.



                  Try it online!



                  How it works



                  Anonymous function that implements a recursive filter.



                  'cbaa'-98 is a shorter form to produce [1 0 -1 -1].



                  2:n<5 is a shorter form to produce [1 1 1 0 0 ··· 0] (n−1 terms).



                  filter(1,[1 0 -1 -1],[1 1 1 0 0 ··· 0]) passes the input [1 1 1 0 0 ··· 0] through a discrete-time filter defined by a transfer function with numerator coefficient 1 and denominator coefficients [1 0 -1 -1].






                  share|improve this answer











                  $endgroup$





















                    3












                    $begingroup$


                    Jelly, 11 bytes



                    5B+Ɲ2ị;Ʋ⁸¡Ḣ


                    Try it online!



                    0-indexed.






                    share|improve this answer











                    $endgroup$













                    • $begingroup$
                      Can you specify whether this answer is 0-indexed or 1-indexed?
                      $endgroup$
                      – Tau
                      yesterday










                    • $begingroup$
                      @Tau It's 0-indexed. I've edited it in.
                      $endgroup$
                      – Erik the Outgolfer
                      yesterday





















                    3












                    $begingroup$


                    Perl 6, 24 bytes



                    {(1,1,1,*+*+!*...*)[$_]}


                    Try it online!



                    A pretty standard generated sequence, with each new element generated by the expression * + * + !*. That adds the third-previous element, the second-previous element, and the logical negation of the previous element, which is always False, which is numerically zero.






                    share|improve this answer











                    $endgroup$













                    • $begingroup$
                      Why is this community wiki?
                      $endgroup$
                      – Jo King
                      yesterday










                    • $begingroup$
                      @JoKing Beats me. If I did it somehow, it wasn't on purpose.
                      $endgroup$
                      – Sean
                      12 hours ago



















                    3












                    $begingroup$


                    Lua 5.3, 49 48 bytes





                    function f(n)return n<4 and 1or f(n-2)+f(n-3)end


                    Try it online!



                    Vanilla Lua doesn't have coercion of booleans to strings (even tonumber(true) returns nil), so you have to use a pseudo-ternary operator. This version is 1-indexed, like all of Lua. The 1or part has to be changed to 1 or in Lua 5.1, which has a different way of lexing numbers.






                    share|improve this answer











                    $endgroup$





















                      3












                      $begingroup$


                      APL (Dyalog Unicode), 20 18 17 bytesSBCS



                      This code is 1-indexed. It's the same number of bytes to get n items of the Padovan sequence, as you have to drop the last few extra members. It's also the same number of bytes to get 0-indexing.



                      Edit: -2 bytes thanks to ngn. -1 byte thanks to ngn



                      4⌷2(⊢,⍨2⌷+/)⍣⎕×⍳3


                      Try it online!



                      Explanation



                      4⌷2(⊢,⍨2⌷+/)⍣⎕×⍳3

                      ⍺(. . . .)⍣⎕⍵ This format simply takes the input ⎕ and applies the function
                      inside the brackets (...) to its operands (here marked ⍵ and ⍺).
                      2(. . .+/)⍣⎕×⍳3 In this case, our ⍵, the left argument, is the array 1 1 1,
                      where we save our results as the function is repeatedly applied
                      and our ⍺, 2, is our right argument and is immediately applied to +/,
                      so that we have 2+/ which will return the pairwise sums of our array.
                      2⌷ We take the second pairwise sum, f(n-2) + f(n-3)
                      ⊢,⍨ And add it to the head of our array.
                      4⌷ When we've finished adding Padovan numbers to the end of our list,
                      the n-th Padovan number (1-indexed) is the 4th member of that list,
                      and so, we implicitly return that.





                      share|improve this answer











                      $endgroup$





















                        3












                        $begingroup$


                        K (ngn/k), 24 20 bytes



                        -4 bytes thanks to ngn!



                        {$[x<3;1;+/o'x-2 3]}


                        Try it online!



                        0-indexed, first N terms






                        share|improve this answer











                        $endgroup$









                        • 1




                          $begingroup$
                          f[x-2]+f[x-3] -> +/o'x-2 3 (o is "recur")
                          $endgroup$
                          – ngn
                          11 hours ago










                        • $begingroup$
                          @ngn Thanks! I tried it (without success) in J; it's elegant here.
                          $endgroup$
                          – Galen Ivanov
                          10 hours ago










                        • $begingroup$
                          @ngn In fact here's one possibillity how it looks in J: (1#.2 3$:@-~])`1:@.(3&>)
                          $endgroup$
                          – Galen Ivanov
                          9 hours ago










                        • $begingroup$
                          ah, right, base-1 decode is a train-friendly way to sum :)
                          $endgroup$
                          – ngn
                          9 hours ago






                        • 2




                          $begingroup$
                          1: -> # in the j solution
                          $endgroup$
                          – ngn
                          9 hours ago





















                        2












                        $begingroup$

                        JavaScript (ES6), 23 bytes



                        Implements the recursive definition of A000931. Returns the $N$th term, 0-indexed.





                        f=n=>n<3||f(n-2)+f(n-3)


                        Try it online!






                        share|improve this answer











                        $endgroup$













                        • $begingroup$
                          I don't think it's reasonable to say that returning true is the same as returning 1 if the rest of the output is numbers.
                          $endgroup$
                          – Nit
                          17 hours ago










                        • $begingroup$
                          @Nit Relevant meta post.
                          $endgroup$
                          – Arnauld
                          16 hours ago



















                        2












                        $begingroup$


                        Japt -N, 12 bytes



                        <3ªßUµ2 +ß´U


                        Try it






                        share|improve this answer









                        $endgroup$













                        • $begingroup$
                          Looks like 12 is the best we can do :
                          $endgroup$
                          – Shaggy
                          yesterday



















                        2












                        $begingroup$

                        Pyth, 16 bytes



                        L?<b3!b+y-b2y-b3


                        This defines the function y. Try it here!



                        Here's a more fun solution, though it's 9 bytes longer; bytes could be shaved though.



                        +l{sa.pMf.Am&>d2%d2T./QY!


                        This uses the definition given by David Callan on the OEIS page: "a(n) = number of compositions of n into parts that are odd and >= 3." Try it here! It takes input directly instead of defining a function.






                        share|improve this answer









                        $endgroup$













                        • $begingroup$
                          y-b2y-b3 could maybe be refactored with either bifurcate or L? Though declaring an array of 2 elements is costly. yL-Lb2,3 is longer :(
                          $endgroup$
                          – Ven
                          21 hours ago












                        • $begingroup$
                          @Ven I was able to replace +y-b2y-b3 with smy-bdhB2 which is the same amount of bytes; hB2 results in the array [2, 3]
                          $endgroup$
                          – RK.
                          13 hours ago










                        • $begingroup$
                          Well done on hB2. Too bad it's the same byte count.
                          $endgroup$
                          – Ven
                          13 hours ago










                        • $begingroup$
                          Yeah, though I wonder if there's some way to get rid of the d in the map.
                          $endgroup$
                          – RK.
                          13 hours ago



















                        2












                        $begingroup$


                        05AB1E, 8 bytes



                        1Ð)λ£₂₃+


                        Try it online!



                        Bear with me, I haven't golfed in a while. I wonder if there's a shorter substitute for 1Ð) which works in this case (I've tried 1D), 3Å1 etc. but none of them save bytes). Outputs the first $n$ terms of the sequence. Or, without the £, it would output an infinite stream of the terms of the sequence.



                        How?



                        1Ð)λ£₂₃+ | Full program.
                        1Ð) | Initialize the stack with [1, 1, 1].
                        λ | Begin the recursive generation of a list: Starting from some base case,
                        | this command generates an infinite list with the pattern function given.
                        £ | Flag for λ. Instead of outputting an infinite stream, only print the first n.
                        ₂₃+ | Add a(n-2) and a(n-3).





                        share|improve this answer











                        $endgroup$













                        • $begingroup$
                          I don't think 1Ð) can be 2 bytes tbh. I can think of six different 3-bytes alternatives, but no 2-byters.
                          $endgroup$
                          – Kevin Cruijssen
                          19 hours ago



















                        2












                        $begingroup$


                        R + pryr, 38 36 bytes



                        Zero-indexed recursive function.





                        f=pryr::f(`if`(n<3,1,f(n-2)+f(n-3)))


                        Try it online!



                        Thanks to @Giuseppe for pointing out two obviously needless bytes.






                        share|improve this answer











                        $endgroup$









                        • 1




                          $begingroup$
                          If you're going to be using pryr, the language should be R + pryr and this can be 36 bytes
                          $endgroup$
                          – Giuseppe
                          9 hours ago












                        • $begingroup$
                          @Giuseppe thanks! Updated now.
                          $endgroup$
                          – rturnbull
                          8 hours ago



















                        1












                        $begingroup$


                        C# (Visual C# Interactive Compiler), 34 bytes





                        int f(int g)=>g<3?1:f(g-2)+f(g-3);


                        Try it online!






                        share|improve this answer









                        $endgroup$





















                          1












                          $begingroup$

                          TI-BASIC (TI-84), 34 bytes



                          [[0,1,0][0,0,1][1,1,0]]^(Ans+5:Ans(1,1


                          0-indexed $N$th term of the sequence.



                          Input is in Ans.

                          Output is in Ans and is automatically printed out.



                          I figured that enough time had passed, plus multiple answers had been posted, of which there were many which out-golfed this answer.



                          Example:



                          0
                          0
                          prgmCDGFD
                          1
                          9
                          9
                          prgmCDGFD
                          9
                          16
                          16
                          prgmCDGFD
                          65


                          Explanation:



                          [[0,1,0][0,0,1][1,1,0]]^(Ans+5:Ans(1,1      ;full program (example input: 6)

                          [[0,1,0][0,0,1][1,1,0]] ;generate the following matrix:
                          ; [0 1 0]
                          ; [0 0 1]
                          ; [1 1 0]
                          ^(Ans+5 ;then raise it to the power of: input + 5
                          ; [4 7 5]
                          ; [5 9 7]
                          ; [7 12 9]
                          Ans(1,1 ;get the top-left index and leave it in "Ans"
                          ;implicitly print Ans





                          share|improve this answer









                          $endgroup$





















                            1












                            $begingroup$


                            Ruby, 26 bytes





                            f=->n{n<3?1:f[n-2]+f[n-3]}


                            Try it online!






                            share|improve this answer









                            $endgroup$





















                              1












                              $begingroup$


                              Japt, 12 bytes



                              Returns the first n terms, 0-indexed. Replace h with g to return the nth term, 1-indexed.



                              ÈnZs3n)x}hBì


                              Try it



                              ÈnZs3n)x}hBì     :Implicit input of integer U
                              B :11
                              ì :Convert to digit array
                              h :Repeat the following until the length of the array is U, pushing the result to the array each time
                              È : Take the last element X from the array Z and pass it through the following function
                              n : Subtract X from
                              Zs : Slice Z
                              3n : -3, giving the last 3 elements in the array
                              ) : End slice
                              x : Reduce by addition
                              } : End function





                              share|improve this answer











                              $endgroup$





















                                1












                                $begingroup$


                                Perl 5, 34 bytes





                                sub f{"@_"<3||f("@_"-2)+f("@_"-3)}


                                Try it online!






                                share|improve this answer









                                $endgroup$





















                                  1












                                  $begingroup$


                                  C++ (gcc), 81 75 bytes



                                  -6 bytes to small golfing





                                  int a(int n){int a=1,b=1,c=1,d,i=2;for(;i++<n;)d=a+b,a=b,b=c,c=d;return c;}


                                  Try it online!



                                  Simple function to compute the values iteratively. No loop occurs for n<3, so the first cases default to the initial 1.






                                  share|improve this answer











                                  $endgroup$





















                                    1












                                    $begingroup$

                                    Java, 41 bytes



                                    Can't use a lambda (runtime error). Port of this Javascript answer



                                    int f(int n){return n<3?1:f(n-2)+f(n-3);}


                                    TIO






                                    share|improve this answer









                                    $endgroup$





















                                      1












                                      $begingroup$

                                      x86 32-bit machine code, 17 bytes



                                      53 33 db f7 e3 43 83 c1 04 03 d8 93 92 e2 fa 5b c3


                                      Disassembly:



                                      00CE1250 53                   push        ebx  
                                      00CE1251 33 DB xor ebx,ebx
                                      00CE1253 F7 E3 mul eax,ebx
                                      00CE1255 43 inc ebx
                                      00CE1256 83 C1 04 add ecx,4
                                      00CE1259 03 D8 add ebx,eax
                                      00CE125B 93 xchg eax,ebx
                                      00CE125C 92 xchg eax,edx
                                      00CE125D E2 FA loop myloop (0CE1259h)
                                      00CE125F 5B pop ebx
                                      00CE1260 C3 ret


                                      It is 0-indexed. The initialization is conveniently achieved by calculating eax * 0. The 128-bit result is 0, and it goes in edx:eax.



                                      At the beginning of each iteration, the order of the registers is ebx, eax, edx. I had to choose the right order to take advantage of the encoding for the xchg eax instruction - 1 byte.



                                      I had to add 4 to the loop counter in order to let the output reach eax, which holds the function's return value in the fastcall convention.



                                      I could use some other calling convention, which doesn't require saving and restoring ebx, but fastcall is fun anyway :)






                                      share|improve this answer









                                      $endgroup$













                                      • $begingroup$
                                        I love to see machine code answers on PP&CG! +1
                                        $endgroup$
                                        – Tau
                                        3 hours ago



















                                      1












                                      $begingroup$


                                      Wolfram Language (Mathematica), 26 bytes



                                      If[#<3,1,#0[#-2]+#0[#-3]]&


                                      Try it online!






                                      share|improve this answer









                                      $endgroup$

















                                        1 2
                                        next




                                        Your Answer





                                        StackExchange.ifUsing("editor", function () {
                                        return StackExchange.using("mathjaxEditing", function () {
                                        StackExchange.MarkdownEditor.creationCallbacks.add(function (editor, postfix) {
                                        StackExchange.mathjaxEditing.prepareWmdForMathJax(editor, postfix, [["\$", "\$"]]);
                                        });
                                        });
                                        }, "mathjax-editing");

                                        StackExchange.ifUsing("editor", function () {
                                        StackExchange.using("externalEditor", function () {
                                        StackExchange.using("snippets", function () {
                                        StackExchange.snippets.init();
                                        });
                                        });
                                        }, "code-snippets");

                                        StackExchange.ready(function() {
                                        var channelOptions = {
                                        tags: "".split(" "),
                                        id: "200"
                                        };
                                        initTagRenderer("".split(" "), "".split(" "), channelOptions);

                                        StackExchange.using("externalEditor", function() {
                                        // Have to fire editor after snippets, if snippets enabled
                                        if (StackExchange.settings.snippets.snippetsEnabled) {
                                        StackExchange.using("snippets", function() {
                                        createEditor();
                                        });
                                        }
                                        else {
                                        createEditor();
                                        }
                                        });

                                        function createEditor() {
                                        StackExchange.prepareEditor({
                                        heartbeatType: 'answer',
                                        autoActivateHeartbeat: false,
                                        convertImagesToLinks: false,
                                        noModals: true,
                                        showLowRepImageUploadWarning: true,
                                        reputationToPostImages: null,
                                        bindNavPrevention: true,
                                        postfix: "",
                                        imageUploader: {
                                        brandingHtml: "Powered by u003ca class="icon-imgur-white" href="https://imgur.com/"u003eu003c/au003e",
                                        contentPolicyHtml: "User contributions licensed under u003ca href="https://creativecommons.org/licenses/by-sa/3.0/"u003ecc by-sa 3.0 with attribution requiredu003c/au003e u003ca href="https://stackoverflow.com/legal/content-policy"u003e(content policy)u003c/au003e",
                                        allowUrls: true
                                        },
                                        onDemand: true,
                                        discardSelector: ".discard-answer"
                                        ,immediatelyShowMarkdownHelp:true
                                        });


                                        }
                                        });














                                        draft saved

                                        draft discarded


















                                        StackExchange.ready(
                                        function () {
                                        StackExchange.openid.initPostLogin('.new-post-login', 'https%3a%2f%2fcodegolf.stackexchange.com%2fquestions%2f182797%2fpatience-young-padovan%23new-answer', 'question_page');
                                        }
                                        );

                                        Post as a guest















                                        Required, but never shown

























                                        32 Answers
                                        32






                                        active

                                        oldest

                                        votes








                                        32 Answers
                                        32






                                        active

                                        oldest

                                        votes









                                        active

                                        oldest

                                        votes






                                        active

                                        oldest

                                        votes








                                        1 2
                                        next










                                        36












                                        $begingroup$


                                        Jelly, 10 bytes



                                        9s3’Ẓæ*³FṀ


                                        Try it online!



                                        1-indexed. Computes the largest element of: $$begin{bmatrix}0&0&1 \ 1&0&1 \ 0&1&0end{bmatrix}^n$$
                                        where the binary matrix is conveniently computed as: $$begin{bmatrix}mathsf{isprime}(0)&mathsf{isprime}(1)&mathsf{isprime}(2) \ mathsf{isprime}(3)&mathsf{isprime}(4)&mathsf{isprime}(5) \ mathsf{isprime}(6)&mathsf{isprime}(7)&mathsf{isprime}(8)end{bmatrix}$$



                                        (this is a total coincidence.)



                                        9s3         [[1,2,3],[4,5,6],[7,8,9]]    9 split 3
                                        ’ [[0,1,2],[3,4,5],[6,7,8]] decrease
                                        Ẓ [[0,0,1],[1,0,1],[0,1,0]] isprime
                                        æ*³ [[0,0,1],[1,0,1],[0,1,0]]^n matrix power by input
                                        FṀ flatten, maximum





                                        share|improve this answer









                                        $endgroup$









                                        • 17




                                          $begingroup$
                                          this is clearly some kind of voodoo
                                          $endgroup$
                                          – Pureferret
                                          18 hours ago






                                        • 6




                                          $begingroup$
                                          This should be published.
                                          $endgroup$
                                          – YSC
                                          17 hours ago






                                        • 2




                                          $begingroup$
                                          @YSC It has already been published in A000931. I'd never have guess the primes trick:)
                                          $endgroup$
                                          – flawr
                                          14 hours ago










                                        • $begingroup$
                                          @flawr ho... I missed it
                                          $endgroup$
                                          – YSC
                                          13 hours ago






                                        • 1




                                          $begingroup$
                                          ...make that "unless someone can golf two bytes off this one" :) (now that I have a 9 byter)
                                          $endgroup$
                                          – Jonathan Allan
                                          12 hours ago


















                                        36












                                        $begingroup$


                                        Jelly, 10 bytes



                                        9s3’Ẓæ*³FṀ


                                        Try it online!



                                        1-indexed. Computes the largest element of: $$begin{bmatrix}0&0&1 \ 1&0&1 \ 0&1&0end{bmatrix}^n$$
                                        where the binary matrix is conveniently computed as: $$begin{bmatrix}mathsf{isprime}(0)&mathsf{isprime}(1)&mathsf{isprime}(2) \ mathsf{isprime}(3)&mathsf{isprime}(4)&mathsf{isprime}(5) \ mathsf{isprime}(6)&mathsf{isprime}(7)&mathsf{isprime}(8)end{bmatrix}$$



                                        (this is a total coincidence.)



                                        9s3         [[1,2,3],[4,5,6],[7,8,9]]    9 split 3
                                        ’ [[0,1,2],[3,4,5],[6,7,8]] decrease
                                        Ẓ [[0,0,1],[1,0,1],[0,1,0]] isprime
                                        æ*³ [[0,0,1],[1,0,1],[0,1,0]]^n matrix power by input
                                        FṀ flatten, maximum





                                        share|improve this answer









                                        $endgroup$









                                        • 17




                                          $begingroup$
                                          this is clearly some kind of voodoo
                                          $endgroup$
                                          – Pureferret
                                          18 hours ago






                                        • 6




                                          $begingroup$
                                          This should be published.
                                          $endgroup$
                                          – YSC
                                          17 hours ago






                                        • 2




                                          $begingroup$
                                          @YSC It has already been published in A000931. I'd never have guess the primes trick:)
                                          $endgroup$
                                          – flawr
                                          14 hours ago










                                        • $begingroup$
                                          @flawr ho... I missed it
                                          $endgroup$
                                          – YSC
                                          13 hours ago






                                        • 1




                                          $begingroup$
                                          ...make that "unless someone can golf two bytes off this one" :) (now that I have a 9 byter)
                                          $endgroup$
                                          – Jonathan Allan
                                          12 hours ago
















                                        36












                                        36








                                        36





                                        $begingroup$


                                        Jelly, 10 bytes



                                        9s3’Ẓæ*³FṀ


                                        Try it online!



                                        1-indexed. Computes the largest element of: $$begin{bmatrix}0&0&1 \ 1&0&1 \ 0&1&0end{bmatrix}^n$$
                                        where the binary matrix is conveniently computed as: $$begin{bmatrix}mathsf{isprime}(0)&mathsf{isprime}(1)&mathsf{isprime}(2) \ mathsf{isprime}(3)&mathsf{isprime}(4)&mathsf{isprime}(5) \ mathsf{isprime}(6)&mathsf{isprime}(7)&mathsf{isprime}(8)end{bmatrix}$$



                                        (this is a total coincidence.)



                                        9s3         [[1,2,3],[4,5,6],[7,8,9]]    9 split 3
                                        ’ [[0,1,2],[3,4,5],[6,7,8]] decrease
                                        Ẓ [[0,0,1],[1,0,1],[0,1,0]] isprime
                                        æ*³ [[0,0,1],[1,0,1],[0,1,0]]^n matrix power by input
                                        FṀ flatten, maximum





                                        share|improve this answer









                                        $endgroup$




                                        Jelly, 10 bytes



                                        9s3’Ẓæ*³FṀ


                                        Try it online!



                                        1-indexed. Computes the largest element of: $$begin{bmatrix}0&0&1 \ 1&0&1 \ 0&1&0end{bmatrix}^n$$
                                        where the binary matrix is conveniently computed as: $$begin{bmatrix}mathsf{isprime}(0)&mathsf{isprime}(1)&mathsf{isprime}(2) \ mathsf{isprime}(3)&mathsf{isprime}(4)&mathsf{isprime}(5) \ mathsf{isprime}(6)&mathsf{isprime}(7)&mathsf{isprime}(8)end{bmatrix}$$



                                        (this is a total coincidence.)



                                        9s3         [[1,2,3],[4,5,6],[7,8,9]]    9 split 3
                                        ’ [[0,1,2],[3,4,5],[6,7,8]] decrease
                                        Ẓ [[0,0,1],[1,0,1],[0,1,0]] isprime
                                        æ*³ [[0,0,1],[1,0,1],[0,1,0]]^n matrix power by input
                                        FṀ flatten, maximum






                                        share|improve this answer












                                        share|improve this answer



                                        share|improve this answer










                                        answered yesterday









                                        LynnLynn

                                        50.5k898233




                                        50.5k898233








                                        • 17




                                          $begingroup$
                                          this is clearly some kind of voodoo
                                          $endgroup$
                                          – Pureferret
                                          18 hours ago






                                        • 6




                                          $begingroup$
                                          This should be published.
                                          $endgroup$
                                          – YSC
                                          17 hours ago






                                        • 2




                                          $begingroup$
                                          @YSC It has already been published in A000931. I'd never have guess the primes trick:)
                                          $endgroup$
                                          – flawr
                                          14 hours ago










                                        • $begingroup$
                                          @flawr ho... I missed it
                                          $endgroup$
                                          – YSC
                                          13 hours ago






                                        • 1




                                          $begingroup$
                                          ...make that "unless someone can golf two bytes off this one" :) (now that I have a 9 byter)
                                          $endgroup$
                                          – Jonathan Allan
                                          12 hours ago
















                                        • 17




                                          $begingroup$
                                          this is clearly some kind of voodoo
                                          $endgroup$
                                          – Pureferret
                                          18 hours ago






                                        • 6




                                          $begingroup$
                                          This should be published.
                                          $endgroup$
                                          – YSC
                                          17 hours ago






                                        • 2




                                          $begingroup$
                                          @YSC It has already been published in A000931. I'd never have guess the primes trick:)
                                          $endgroup$
                                          – flawr
                                          14 hours ago










                                        • $begingroup$
                                          @flawr ho... I missed it
                                          $endgroup$
                                          – YSC
                                          13 hours ago






                                        • 1




                                          $begingroup$
                                          ...make that "unless someone can golf two bytes off this one" :) (now that I have a 9 byter)
                                          $endgroup$
                                          – Jonathan Allan
                                          12 hours ago










                                        17




                                        17




                                        $begingroup$
                                        this is clearly some kind of voodoo
                                        $endgroup$
                                        – Pureferret
                                        18 hours ago




                                        $begingroup$
                                        this is clearly some kind of voodoo
                                        $endgroup$
                                        – Pureferret
                                        18 hours ago




                                        6




                                        6




                                        $begingroup$
                                        This should be published.
                                        $endgroup$
                                        – YSC
                                        17 hours ago




                                        $begingroup$
                                        This should be published.
                                        $endgroup$
                                        – YSC
                                        17 hours ago




                                        2




                                        2




                                        $begingroup$
                                        @YSC It has already been published in A000931. I'd never have guess the primes trick:)
                                        $endgroup$
                                        – flawr
                                        14 hours ago




                                        $begingroup$
                                        @YSC It has already been published in A000931. I'd never have guess the primes trick:)
                                        $endgroup$
                                        – flawr
                                        14 hours ago












                                        $begingroup$
                                        @flawr ho... I missed it
                                        $endgroup$
                                        – YSC
                                        13 hours ago




                                        $begingroup$
                                        @flawr ho... I missed it
                                        $endgroup$
                                        – YSC
                                        13 hours ago




                                        1




                                        1




                                        $begingroup$
                                        ...make that "unless someone can golf two bytes off this one" :) (now that I have a 9 byter)
                                        $endgroup$
                                        – Jonathan Allan
                                        12 hours ago






                                        $begingroup$
                                        ...make that "unless someone can golf two bytes off this one" :) (now that I have a 9 byter)
                                        $endgroup$
                                        – Jonathan Allan
                                        12 hours ago













                                        19












                                        $begingroup$


                                        Oasis, 5 bytes



                                        nth term 0-indexed



                                        cd+1V


                                        Try it online!



                                        Explanation



                                           1V   # a(0) = 1
                                        # a(1) = 1
                                        # a(2) = 1
                                        # a(n) =
                                        c # a(n-2)
                                        + # +
                                        d # a(n-3)





                                        share|improve this answer









                                        $endgroup$


















                                          19












                                          $begingroup$


                                          Oasis, 5 bytes



                                          nth term 0-indexed



                                          cd+1V


                                          Try it online!



                                          Explanation



                                             1V   # a(0) = 1
                                          # a(1) = 1
                                          # a(2) = 1
                                          # a(n) =
                                          c # a(n-2)
                                          + # +
                                          d # a(n-3)





                                          share|improve this answer









                                          $endgroup$
















                                            19












                                            19








                                            19





                                            $begingroup$


                                            Oasis, 5 bytes



                                            nth term 0-indexed



                                            cd+1V


                                            Try it online!



                                            Explanation



                                               1V   # a(0) = 1
                                            # a(1) = 1
                                            # a(2) = 1
                                            # a(n) =
                                            c # a(n-2)
                                            + # +
                                            d # a(n-3)





                                            share|improve this answer









                                            $endgroup$




                                            Oasis, 5 bytes



                                            nth term 0-indexed



                                            cd+1V


                                            Try it online!



                                            Explanation



                                               1V   # a(0) = 1
                                            # a(1) = 1
                                            # a(2) = 1
                                            # a(n) =
                                            c # a(n-2)
                                            + # +
                                            d # a(n-3)






                                            share|improve this answer












                                            share|improve this answer



                                            share|improve this answer










                                            answered yesterday









                                            EmignaEmigna

                                            47.6k433145




                                            47.6k433145























                                                16












                                                $begingroup$


                                                Haskell, 26 bytes





                                                (l!!)
                                                l=1:1:1:2:scanl(+)2l


                                                Try it online! Outputs the n'th term zero-indexed.



                                                I thought that the "obvious" recursive solution below would be unbeatable, but then I found this. It's similar to the classic golfy expression l=1:scanl(+)1l for the infinite Fibonacci list, but here the difference between adjacent elements is the term 4 positions back. We can more directly write l=1:1:zipWith(+)l(0:l), but that's longer.



                                                If this challenge allowed infinite list output, we could cut the first line and have 20 bytes.



                                                27 bytes





                                                f n|n<3=1|1>0=f(n-2)+f(n-3)


                                                Try it online!






                                                share|improve this answer









                                                $endgroup$


















                                                  16












                                                  $begingroup$


                                                  Haskell, 26 bytes





                                                  (l!!)
                                                  l=1:1:1:2:scanl(+)2l


                                                  Try it online! Outputs the n'th term zero-indexed.



                                                  I thought that the "obvious" recursive solution below would be unbeatable, but then I found this. It's similar to the classic golfy expression l=1:scanl(+)1l for the infinite Fibonacci list, but here the difference between adjacent elements is the term 4 positions back. We can more directly write l=1:1:zipWith(+)l(0:l), but that's longer.



                                                  If this challenge allowed infinite list output, we could cut the first line and have 20 bytes.



                                                  27 bytes





                                                  f n|n<3=1|1>0=f(n-2)+f(n-3)


                                                  Try it online!






                                                  share|improve this answer









                                                  $endgroup$
















                                                    16












                                                    16








                                                    16





                                                    $begingroup$


                                                    Haskell, 26 bytes





                                                    (l!!)
                                                    l=1:1:1:2:scanl(+)2l


                                                    Try it online! Outputs the n'th term zero-indexed.



                                                    I thought that the "obvious" recursive solution below would be unbeatable, but then I found this. It's similar to the classic golfy expression l=1:scanl(+)1l for the infinite Fibonacci list, but here the difference between adjacent elements is the term 4 positions back. We can more directly write l=1:1:zipWith(+)l(0:l), but that's longer.



                                                    If this challenge allowed infinite list output, we could cut the first line and have 20 bytes.



                                                    27 bytes





                                                    f n|n<3=1|1>0=f(n-2)+f(n-3)


                                                    Try it online!






                                                    share|improve this answer









                                                    $endgroup$




                                                    Haskell, 26 bytes





                                                    (l!!)
                                                    l=1:1:1:2:scanl(+)2l


                                                    Try it online! Outputs the n'th term zero-indexed.



                                                    I thought that the "obvious" recursive solution below would be unbeatable, but then I found this. It's similar to the classic golfy expression l=1:scanl(+)1l for the infinite Fibonacci list, but here the difference between adjacent elements is the term 4 positions back. We can more directly write l=1:1:zipWith(+)l(0:l), but that's longer.



                                                    If this challenge allowed infinite list output, we could cut the first line and have 20 bytes.



                                                    27 bytes





                                                    f n|n<3=1|1>0=f(n-2)+f(n-3)


                                                    Try it online!







                                                    share|improve this answer












                                                    share|improve this answer



                                                    share|improve this answer










                                                    answered yesterday









                                                    xnorxnor

                                                    93.6k18190450




                                                    93.6k18190450























                                                        14












                                                        $begingroup$


                                                        Jelly,  10 9  8 bytes



                                                        ŻṚm2Jc$S


                                                        A monadic Link accepting n (0-indexed) which yields P(n).



                                                        Try it online!



                                                        How?



                                                        Implements $P(n) = sum_{i=0}^{lfloorfrac{n}2rfloor}binom{i+1}{n-2i}$



                                                        ŻṚm2Jc$S - Link: integer, n       e.g. 20
                                                        Ż - zero range [0, 1, 2, 3, 4, ..., 19, 20]
                                                        Ṛ - reverse [20, 19, ..., 4, 3, 2, 1, 0]
                                                        m2 - modulo-slice with 2 [20, 18, 16, 14, 12, 10, 8, 6, 4, 2, 0] <- n-2i
                                                        $ - last two links as a monad:
                                                        J - range of length [ 1, 2, 3, 4, 5, 6, 7, 8, 9, 10, 11] <- i+1
                                                        c - left-choose-right [ 0, 0, 0, 0, 0, 0, 0, 28,126, 45, 1]
                                                        S - sum 200




                                                        And here is a "twofer"

                                                        ...a totally different method also for 8 bytes (this one is 1-indexed, but much slower):



                                                        3ḊṗRẎ§ċ‘ - Link: n
                                                        3Ḋ - 3 dequeued = [2,3]
                                                        R - range = [1,2,3,...,n]
                                                        ṗ - Cartesian power [[[2],[3]],[[2,2],[2,3],[3,2],[3,3]],[[2,2,2],...],...]
                                                        Ẏ - tighten [[2],[3],[2,2],[2,3],[3,2],[3,3],[2,2,2],...]
                                                        § - sums [ 2, 3, 4, 5, 5, 6, 6,...]
                                                        ‘ - increment n+1
                                                        ċ - count occurrences P(n)





                                                        share|improve this answer











                                                        $endgroup$


















                                                          14












                                                          $begingroup$


                                                          Jelly,  10 9  8 bytes



                                                          ŻṚm2Jc$S


                                                          A monadic Link accepting n (0-indexed) which yields P(n).



                                                          Try it online!



                                                          How?



                                                          Implements $P(n) = sum_{i=0}^{lfloorfrac{n}2rfloor}binom{i+1}{n-2i}$



                                                          ŻṚm2Jc$S - Link: integer, n       e.g. 20
                                                          Ż - zero range [0, 1, 2, 3, 4, ..., 19, 20]
                                                          Ṛ - reverse [20, 19, ..., 4, 3, 2, 1, 0]
                                                          m2 - modulo-slice with 2 [20, 18, 16, 14, 12, 10, 8, 6, 4, 2, 0] <- n-2i
                                                          $ - last two links as a monad:
                                                          J - range of length [ 1, 2, 3, 4, 5, 6, 7, 8, 9, 10, 11] <- i+1
                                                          c - left-choose-right [ 0, 0, 0, 0, 0, 0, 0, 28,126, 45, 1]
                                                          S - sum 200




                                                          And here is a "twofer"

                                                          ...a totally different method also for 8 bytes (this one is 1-indexed, but much slower):



                                                          3ḊṗRẎ§ċ‘ - Link: n
                                                          3Ḋ - 3 dequeued = [2,3]
                                                          R - range = [1,2,3,...,n]
                                                          ṗ - Cartesian power [[[2],[3]],[[2,2],[2,3],[3,2],[3,3]],[[2,2,2],...],...]
                                                          Ẏ - tighten [[2],[3],[2,2],[2,3],[3,2],[3,3],[2,2,2],...]
                                                          § - sums [ 2, 3, 4, 5, 5, 6, 6,...]
                                                          ‘ - increment n+1
                                                          ċ - count occurrences P(n)





                                                          share|improve this answer











                                                          $endgroup$
















                                                            14












                                                            14








                                                            14





                                                            $begingroup$


                                                            Jelly,  10 9  8 bytes



                                                            ŻṚm2Jc$S


                                                            A monadic Link accepting n (0-indexed) which yields P(n).



                                                            Try it online!



                                                            How?



                                                            Implements $P(n) = sum_{i=0}^{lfloorfrac{n}2rfloor}binom{i+1}{n-2i}$



                                                            ŻṚm2Jc$S - Link: integer, n       e.g. 20
                                                            Ż - zero range [0, 1, 2, 3, 4, ..., 19, 20]
                                                            Ṛ - reverse [20, 19, ..., 4, 3, 2, 1, 0]
                                                            m2 - modulo-slice with 2 [20, 18, 16, 14, 12, 10, 8, 6, 4, 2, 0] <- n-2i
                                                            $ - last two links as a monad:
                                                            J - range of length [ 1, 2, 3, 4, 5, 6, 7, 8, 9, 10, 11] <- i+1
                                                            c - left-choose-right [ 0, 0, 0, 0, 0, 0, 0, 28,126, 45, 1]
                                                            S - sum 200




                                                            And here is a "twofer"

                                                            ...a totally different method also for 8 bytes (this one is 1-indexed, but much slower):



                                                            3ḊṗRẎ§ċ‘ - Link: n
                                                            3Ḋ - 3 dequeued = [2,3]
                                                            R - range = [1,2,3,...,n]
                                                            ṗ - Cartesian power [[[2],[3]],[[2,2],[2,3],[3,2],[3,3]],[[2,2,2],...],...]
                                                            Ẏ - tighten [[2],[3],[2,2],[2,3],[3,2],[3,3],[2,2,2],...]
                                                            § - sums [ 2, 3, 4, 5, 5, 6, 6,...]
                                                            ‘ - increment n+1
                                                            ċ - count occurrences P(n)





                                                            share|improve this answer











                                                            $endgroup$




                                                            Jelly,  10 9  8 bytes



                                                            ŻṚm2Jc$S


                                                            A monadic Link accepting n (0-indexed) which yields P(n).



                                                            Try it online!



                                                            How?



                                                            Implements $P(n) = sum_{i=0}^{lfloorfrac{n}2rfloor}binom{i+1}{n-2i}$



                                                            ŻṚm2Jc$S - Link: integer, n       e.g. 20
                                                            Ż - zero range [0, 1, 2, 3, 4, ..., 19, 20]
                                                            Ṛ - reverse [20, 19, ..., 4, 3, 2, 1, 0]
                                                            m2 - modulo-slice with 2 [20, 18, 16, 14, 12, 10, 8, 6, 4, 2, 0] <- n-2i
                                                            $ - last two links as a monad:
                                                            J - range of length [ 1, 2, 3, 4, 5, 6, 7, 8, 9, 10, 11] <- i+1
                                                            c - left-choose-right [ 0, 0, 0, 0, 0, 0, 0, 28,126, 45, 1]
                                                            S - sum 200




                                                            And here is a "twofer"

                                                            ...a totally different method also for 8 bytes (this one is 1-indexed, but much slower):



                                                            3ḊṗRẎ§ċ‘ - Link: n
                                                            3Ḋ - 3 dequeued = [2,3]
                                                            R - range = [1,2,3,...,n]
                                                            ṗ - Cartesian power [[[2],[3]],[[2,2],[2,3],[3,2],[3,3]],[[2,2,2],...],...]
                                                            Ẏ - tighten [[2],[3],[2,2],[2,3],[3,2],[3,3],[2,2,2],...]
                                                            § - sums [ 2, 3, 4, 5, 5, 6, 6,...]
                                                            ‘ - increment n+1
                                                            ċ - count occurrences P(n)






                                                            share|improve this answer














                                                            share|improve this answer



                                                            share|improve this answer








                                                            edited 5 hours ago

























                                                            answered yesterday









                                                            Jonathan AllanJonathan Allan

                                                            54k536174




                                                            54k536174























                                                                7












                                                                $begingroup$


                                                                Python 2, 30 bytes





                                                                f=lambda n:n<3or f(n-2)+f(n-3)


                                                                Try it online!



                                                                Returns the n'th term zero indexed. Outputs True for 1.






                                                                share|improve this answer











                                                                $endgroup$


















                                                                  7












                                                                  $begingroup$


                                                                  Python 2, 30 bytes





                                                                  f=lambda n:n<3or f(n-2)+f(n-3)


                                                                  Try it online!



                                                                  Returns the n'th term zero indexed. Outputs True for 1.






                                                                  share|improve this answer











                                                                  $endgroup$
















                                                                    7












                                                                    7








                                                                    7





                                                                    $begingroup$


                                                                    Python 2, 30 bytes





                                                                    f=lambda n:n<3or f(n-2)+f(n-3)


                                                                    Try it online!



                                                                    Returns the n'th term zero indexed. Outputs True for 1.






                                                                    share|improve this answer











                                                                    $endgroup$




                                                                    Python 2, 30 bytes





                                                                    f=lambda n:n<3or f(n-2)+f(n-3)


                                                                    Try it online!



                                                                    Returns the n'th term zero indexed. Outputs True for 1.







                                                                    share|improve this answer














                                                                    share|improve this answer



                                                                    share|improve this answer








                                                                    edited yesterday

























                                                                    answered yesterday









                                                                    xnorxnor

                                                                    93.6k18190450




                                                                    93.6k18190450























                                                                        5












                                                                        $begingroup$


                                                                        Wolfram Language (Mathematica), 33 bytes



                                                                        a@0=a@1=a@2=1;a@n_:=a[n-2]+a[n-3]   


                                                                        1-indexed, returns the nth term



                                                                        Try it online!






                                                                        share|improve this answer









                                                                        $endgroup$


















                                                                          5












                                                                          $begingroup$


                                                                          Wolfram Language (Mathematica), 33 bytes



                                                                          a@0=a@1=a@2=1;a@n_:=a[n-2]+a[n-3]   


                                                                          1-indexed, returns the nth term



                                                                          Try it online!






                                                                          share|improve this answer









                                                                          $endgroup$
















                                                                            5












                                                                            5








                                                                            5





                                                                            $begingroup$


                                                                            Wolfram Language (Mathematica), 33 bytes



                                                                            a@0=a@1=a@2=1;a@n_:=a[n-2]+a[n-3]   


                                                                            1-indexed, returns the nth term



                                                                            Try it online!






                                                                            share|improve this answer









                                                                            $endgroup$




                                                                            Wolfram Language (Mathematica), 33 bytes



                                                                            a@0=a@1=a@2=1;a@n_:=a[n-2]+a[n-3]   


                                                                            1-indexed, returns the nth term



                                                                            Try it online!







                                                                            share|improve this answer












                                                                            share|improve this answer



                                                                            share|improve this answer










                                                                            answered yesterday









                                                                            J42161217J42161217

                                                                            13.9k21353




                                                                            13.9k21353























                                                                                5












                                                                                $begingroup$


                                                                                J, 23 bytes



                                                                                -1 byte thanks to ngn and Galen



                                                                                closed form, 26 bytes



                                                                                0.5<.@+1.04535%~1.32472^<:


                                                                                Try it online!



                                                                                iterative, 23 bytes



                                                                                (],1#._2 _3{ ::1:])^:[#


                                                                                Try it online!






                                                                                share|improve this answer











                                                                                $endgroup$









                                                                                • 1




                                                                                  $begingroup$
                                                                                  Another 24-byte solution (boring) : (1#.2 3$:@-~])`1:@.(3&>) Try it online!
                                                                                  $endgroup$
                                                                                  – Galen Ivanov
                                                                                  9 hours ago












                                                                                • $begingroup$
                                                                                  23 bytes thanks to ngn 1: -> # : Try it online!
                                                                                  $endgroup$
                                                                                  – Galen Ivanov
                                                                                  9 hours ago












                                                                                • $begingroup$
                                                                                  @GalenIvanov tyvm, that's a great trick.
                                                                                  $endgroup$
                                                                                  – Jonah
                                                                                  8 hours ago
















                                                                                5












                                                                                $begingroup$


                                                                                J, 23 bytes



                                                                                -1 byte thanks to ngn and Galen



                                                                                closed form, 26 bytes



                                                                                0.5<.@+1.04535%~1.32472^<:


                                                                                Try it online!



                                                                                iterative, 23 bytes



                                                                                (],1#._2 _3{ ::1:])^:[#


                                                                                Try it online!






                                                                                share|improve this answer











                                                                                $endgroup$









                                                                                • 1




                                                                                  $begingroup$
                                                                                  Another 24-byte solution (boring) : (1#.2 3$:@-~])`1:@.(3&>) Try it online!
                                                                                  $endgroup$
                                                                                  – Galen Ivanov
                                                                                  9 hours ago












                                                                                • $begingroup$
                                                                                  23 bytes thanks to ngn 1: -> # : Try it online!
                                                                                  $endgroup$
                                                                                  – Galen Ivanov
                                                                                  9 hours ago












                                                                                • $begingroup$
                                                                                  @GalenIvanov tyvm, that's a great trick.
                                                                                  $endgroup$
                                                                                  – Jonah
                                                                                  8 hours ago














                                                                                5












                                                                                5








                                                                                5





                                                                                $begingroup$


                                                                                J, 23 bytes



                                                                                -1 byte thanks to ngn and Galen



                                                                                closed form, 26 bytes



                                                                                0.5<.@+1.04535%~1.32472^<:


                                                                                Try it online!



                                                                                iterative, 23 bytes



                                                                                (],1#._2 _3{ ::1:])^:[#


                                                                                Try it online!






                                                                                share|improve this answer











                                                                                $endgroup$




                                                                                J, 23 bytes



                                                                                -1 byte thanks to ngn and Galen



                                                                                closed form, 26 bytes



                                                                                0.5<.@+1.04535%~1.32472^<:


                                                                                Try it online!



                                                                                iterative, 23 bytes



                                                                                (],1#._2 _3{ ::1:])^:[#


                                                                                Try it online!







                                                                                share|improve this answer














                                                                                share|improve this answer



                                                                                share|improve this answer








                                                                                edited 8 hours ago

























                                                                                answered yesterday









                                                                                JonahJonah

                                                                                2,6611017




                                                                                2,6611017








                                                                                • 1




                                                                                  $begingroup$
                                                                                  Another 24-byte solution (boring) : (1#.2 3$:@-~])`1:@.(3&>) Try it online!
                                                                                  $endgroup$
                                                                                  – Galen Ivanov
                                                                                  9 hours ago












                                                                                • $begingroup$
                                                                                  23 bytes thanks to ngn 1: -> # : Try it online!
                                                                                  $endgroup$
                                                                                  – Galen Ivanov
                                                                                  9 hours ago












                                                                                • $begingroup$
                                                                                  @GalenIvanov tyvm, that's a great trick.
                                                                                  $endgroup$
                                                                                  – Jonah
                                                                                  8 hours ago














                                                                                • 1




                                                                                  $begingroup$
                                                                                  Another 24-byte solution (boring) : (1#.2 3$:@-~])`1:@.(3&>) Try it online!
                                                                                  $endgroup$
                                                                                  – Galen Ivanov
                                                                                  9 hours ago












                                                                                • $begingroup$
                                                                                  23 bytes thanks to ngn 1: -> # : Try it online!
                                                                                  $endgroup$
                                                                                  – Galen Ivanov
                                                                                  9 hours ago












                                                                                • $begingroup$
                                                                                  @GalenIvanov tyvm, that's a great trick.
                                                                                  $endgroup$
                                                                                  – Jonah
                                                                                  8 hours ago








                                                                                1




                                                                                1




                                                                                $begingroup$
                                                                                Another 24-byte solution (boring) : (1#.2 3$:@-~])`1:@.(3&>) Try it online!
                                                                                $endgroup$
                                                                                – Galen Ivanov
                                                                                9 hours ago






                                                                                $begingroup$
                                                                                Another 24-byte solution (boring) : (1#.2 3$:@-~])`1:@.(3&>) Try it online!
                                                                                $endgroup$
                                                                                – Galen Ivanov
                                                                                9 hours ago














                                                                                $begingroup$
                                                                                23 bytes thanks to ngn 1: -> # : Try it online!
                                                                                $endgroup$
                                                                                – Galen Ivanov
                                                                                9 hours ago






                                                                                $begingroup$
                                                                                23 bytes thanks to ngn 1: -> # : Try it online!
                                                                                $endgroup$
                                                                                – Galen Ivanov
                                                                                9 hours ago














                                                                                $begingroup$
                                                                                @GalenIvanov tyvm, that's a great trick.
                                                                                $endgroup$
                                                                                – Jonah
                                                                                8 hours ago




                                                                                $begingroup$
                                                                                @GalenIvanov tyvm, that's a great trick.
                                                                                $endgroup$
                                                                                – Jonah
                                                                                8 hours ago











                                                                                4












                                                                                $begingroup$


                                                                                Python 2, 56 48 bytes





                                                                                f=lambda n,a=1,b=1,c=1:n>2and f(n-1,b,c,a+b)or c


                                                                                Try it online!



                                                                                Returns nth value, 0-indexed.






                                                                                share|improve this answer









                                                                                $endgroup$


















                                                                                  4












                                                                                  $begingroup$


                                                                                  Python 2, 56 48 bytes





                                                                                  f=lambda n,a=1,b=1,c=1:n>2and f(n-1,b,c,a+b)or c


                                                                                  Try it online!



                                                                                  Returns nth value, 0-indexed.






                                                                                  share|improve this answer









                                                                                  $endgroup$
















                                                                                    4












                                                                                    4








                                                                                    4





                                                                                    $begingroup$


                                                                                    Python 2, 56 48 bytes





                                                                                    f=lambda n,a=1,b=1,c=1:n>2and f(n-1,b,c,a+b)or c


                                                                                    Try it online!



                                                                                    Returns nth value, 0-indexed.






                                                                                    share|improve this answer









                                                                                    $endgroup$




                                                                                    Python 2, 56 48 bytes





                                                                                    f=lambda n,a=1,b=1,c=1:n>2and f(n-1,b,c,a+b)or c


                                                                                    Try it online!



                                                                                    Returns nth value, 0-indexed.







                                                                                    share|improve this answer












                                                                                    share|improve this answer



                                                                                    share|improve this answer










                                                                                    answered yesterday









                                                                                    Chas BrownChas Brown

                                                                                    5,2191523




                                                                                    5,2191523























                                                                                        4












                                                                                        $begingroup$


                                                                                        Retina, 47 42 bytes



                                                                                        K`0¶1¶0
                                                                                        "$+"+`.+¶(.+)¶.+$
                                                                                        $&¶$.(*_$1*
                                                                                        6,G`


                                                                                        Try it online! Outputs the first n terms on separate lines. Explanation:



                                                                                        K`0¶1¶0


                                                                                        Replace the input with the terms for -2, -1 and 0.



                                                                                        "$+"+`.+¶(.+)¶.+$
                                                                                        $&¶$.(*_$1*


                                                                                        Generate the next n terms using the recurrence relation. *_ here is short for $&*_ which converts the (first) number in the match to unary, while $1* is short for $1*_ which converts the middle number to unary. The $.( returns the decimal sum of its unary arguments, i.e. the sum of the first and middle numbers.



                                                                                        6,G`


                                                                                        Discard the first six characters, i.e. the first three lines.






                                                                                        share|improve this answer











                                                                                        $endgroup$


















                                                                                          4












                                                                                          $begingroup$


                                                                                          Retina, 47 42 bytes



                                                                                          K`0¶1¶0
                                                                                          "$+"+`.+¶(.+)¶.+$
                                                                                          $&¶$.(*_$1*
                                                                                          6,G`


                                                                                          Try it online! Outputs the first n terms on separate lines. Explanation:



                                                                                          K`0¶1¶0


                                                                                          Replace the input with the terms for -2, -1 and 0.



                                                                                          "$+"+`.+¶(.+)¶.+$
                                                                                          $&¶$.(*_$1*


                                                                                          Generate the next n terms using the recurrence relation. *_ here is short for $&*_ which converts the (first) number in the match to unary, while $1* is short for $1*_ which converts the middle number to unary. The $.( returns the decimal sum of its unary arguments, i.e. the sum of the first and middle numbers.



                                                                                          6,G`


                                                                                          Discard the first six characters, i.e. the first three lines.






                                                                                          share|improve this answer











                                                                                          $endgroup$
















                                                                                            4












                                                                                            4








                                                                                            4





                                                                                            $begingroup$


                                                                                            Retina, 47 42 bytes



                                                                                            K`0¶1¶0
                                                                                            "$+"+`.+¶(.+)¶.+$
                                                                                            $&¶$.(*_$1*
                                                                                            6,G`


                                                                                            Try it online! Outputs the first n terms on separate lines. Explanation:



                                                                                            K`0¶1¶0


                                                                                            Replace the input with the terms for -2, -1 and 0.



                                                                                            "$+"+`.+¶(.+)¶.+$
                                                                                            $&¶$.(*_$1*


                                                                                            Generate the next n terms using the recurrence relation. *_ here is short for $&*_ which converts the (first) number in the match to unary, while $1* is short for $1*_ which converts the middle number to unary. The $.( returns the decimal sum of its unary arguments, i.e. the sum of the first and middle numbers.



                                                                                            6,G`


                                                                                            Discard the first six characters, i.e. the first three lines.






                                                                                            share|improve this answer











                                                                                            $endgroup$




                                                                                            Retina, 47 42 bytes



                                                                                            K`0¶1¶0
                                                                                            "$+"+`.+¶(.+)¶.+$
                                                                                            $&¶$.(*_$1*
                                                                                            6,G`


                                                                                            Try it online! Outputs the first n terms on separate lines. Explanation:



                                                                                            K`0¶1¶0


                                                                                            Replace the input with the terms for -2, -1 and 0.



                                                                                            "$+"+`.+¶(.+)¶.+$
                                                                                            $&¶$.(*_$1*


                                                                                            Generate the next n terms using the recurrence relation. *_ here is short for $&*_ which converts the (first) number in the match to unary, while $1* is short for $1*_ which converts the middle number to unary. The $.( returns the decimal sum of its unary arguments, i.e. the sum of the first and middle numbers.



                                                                                            6,G`


                                                                                            Discard the first six characters, i.e. the first three lines.







                                                                                            share|improve this answer














                                                                                            share|improve this answer



                                                                                            share|improve this answer








                                                                                            edited yesterday

























                                                                                            answered yesterday









                                                                                            NeilNeil

                                                                                            82.6k745179




                                                                                            82.6k745179























                                                                                                4












                                                                                                $begingroup$


                                                                                                Cubix, 20 bytes



                                                                                                This is 0 indexed and outputs the Nth term



                                                                                                ;@UOI010+p?/sqq;W.(


                                                                                                Try it online!



                                                                                                Wraps onto a cube with side length 2



                                                                                                    ; @
                                                                                                U O
                                                                                                I 0 1 0 + p ? /
                                                                                                s q q ; W . (
                                                                                                . .
                                                                                                . .


                                                                                                Watch it run





                                                                                                • I010 - Initiates the stack


                                                                                                • +p? - Adds the top of stack, pulls the counter from the bottom of stack and tests


                                                                                                • /;UO@ - If counter is 0, reflect onto top face, remove TOS, u-turn, output and halt


                                                                                                • (sqq;W - If counter is positive, reflect, decrement counter, swap TOS, push top to bottom twice, remove TOS and shift lane back into the main loop.






                                                                                                share|improve this answer









                                                                                                $endgroup$


















                                                                                                  4












                                                                                                  $begingroup$


                                                                                                  Cubix, 20 bytes



                                                                                                  This is 0 indexed and outputs the Nth term



                                                                                                  ;@UOI010+p?/sqq;W.(


                                                                                                  Try it online!



                                                                                                  Wraps onto a cube with side length 2



                                                                                                      ; @
                                                                                                  U O
                                                                                                  I 0 1 0 + p ? /
                                                                                                  s q q ; W . (
                                                                                                  . .
                                                                                                  . .


                                                                                                  Watch it run





                                                                                                  • I010 - Initiates the stack


                                                                                                  • +p? - Adds the top of stack, pulls the counter from the bottom of stack and tests


                                                                                                  • /;UO@ - If counter is 0, reflect onto top face, remove TOS, u-turn, output and halt


                                                                                                  • (sqq;W - If counter is positive, reflect, decrement counter, swap TOS, push top to bottom twice, remove TOS and shift lane back into the main loop.






                                                                                                  share|improve this answer









                                                                                                  $endgroup$
















                                                                                                    4












                                                                                                    4








                                                                                                    4





                                                                                                    $begingroup$


                                                                                                    Cubix, 20 bytes



                                                                                                    This is 0 indexed and outputs the Nth term



                                                                                                    ;@UOI010+p?/sqq;W.(


                                                                                                    Try it online!



                                                                                                    Wraps onto a cube with side length 2



                                                                                                        ; @
                                                                                                    U O
                                                                                                    I 0 1 0 + p ? /
                                                                                                    s q q ; W . (
                                                                                                    . .
                                                                                                    . .


                                                                                                    Watch it run





                                                                                                    • I010 - Initiates the stack


                                                                                                    • +p? - Adds the top of stack, pulls the counter from the bottom of stack and tests


                                                                                                    • /;UO@ - If counter is 0, reflect onto top face, remove TOS, u-turn, output and halt


                                                                                                    • (sqq;W - If counter is positive, reflect, decrement counter, swap TOS, push top to bottom twice, remove TOS and shift lane back into the main loop.






                                                                                                    share|improve this answer









                                                                                                    $endgroup$




                                                                                                    Cubix, 20 bytes



                                                                                                    This is 0 indexed and outputs the Nth term



                                                                                                    ;@UOI010+p?/sqq;W.(


                                                                                                    Try it online!



                                                                                                    Wraps onto a cube with side length 2



                                                                                                        ; @
                                                                                                    U O
                                                                                                    I 0 1 0 + p ? /
                                                                                                    s q q ; W . (
                                                                                                    . .
                                                                                                    . .


                                                                                                    Watch it run





                                                                                                    • I010 - Initiates the stack


                                                                                                    • +p? - Adds the top of stack, pulls the counter from the bottom of stack and tests


                                                                                                    • /;UO@ - If counter is 0, reflect onto top face, remove TOS, u-turn, output and halt


                                                                                                    • (sqq;W - If counter is positive, reflect, decrement counter, swap TOS, push top to bottom twice, remove TOS and shift lane back into the main loop.







                                                                                                    share|improve this answer












                                                                                                    share|improve this answer



                                                                                                    share|improve this answer










                                                                                                    answered yesterday









                                                                                                    MickyTMickyT

                                                                                                    10.3k21637




                                                                                                    10.3k21637























                                                                                                        4












                                                                                                        $begingroup$


                                                                                                        Octave / MATLAB, 35 33 bytes





                                                                                                        @(n)[1 filter(1,'cbaa'-98,2:n<5)]


                                                                                                        Outputs the first n terms.



                                                                                                        Try it online!



                                                                                                        How it works



                                                                                                        Anonymous function that implements a recursive filter.



                                                                                                        'cbaa'-98 is a shorter form to produce [1 0 -1 -1].



                                                                                                        2:n<5 is a shorter form to produce [1 1 1 0 0 ··· 0] (n−1 terms).



                                                                                                        filter(1,[1 0 -1 -1],[1 1 1 0 0 ··· 0]) passes the input [1 1 1 0 0 ··· 0] through a discrete-time filter defined by a transfer function with numerator coefficient 1 and denominator coefficients [1 0 -1 -1].






                                                                                                        share|improve this answer











                                                                                                        $endgroup$


















                                                                                                          4












                                                                                                          $begingroup$


                                                                                                          Octave / MATLAB, 35 33 bytes





                                                                                                          @(n)[1 filter(1,'cbaa'-98,2:n<5)]


                                                                                                          Outputs the first n terms.



                                                                                                          Try it online!



                                                                                                          How it works



                                                                                                          Anonymous function that implements a recursive filter.



                                                                                                          'cbaa'-98 is a shorter form to produce [1 0 -1 -1].



                                                                                                          2:n<5 is a shorter form to produce [1 1 1 0 0 ··· 0] (n−1 terms).



                                                                                                          filter(1,[1 0 -1 -1],[1 1 1 0 0 ··· 0]) passes the input [1 1 1 0 0 ··· 0] through a discrete-time filter defined by a transfer function with numerator coefficient 1 and denominator coefficients [1 0 -1 -1].






                                                                                                          share|improve this answer











                                                                                                          $endgroup$
















                                                                                                            4












                                                                                                            4








                                                                                                            4





                                                                                                            $begingroup$


                                                                                                            Octave / MATLAB, 35 33 bytes





                                                                                                            @(n)[1 filter(1,'cbaa'-98,2:n<5)]


                                                                                                            Outputs the first n terms.



                                                                                                            Try it online!



                                                                                                            How it works



                                                                                                            Anonymous function that implements a recursive filter.



                                                                                                            'cbaa'-98 is a shorter form to produce [1 0 -1 -1].



                                                                                                            2:n<5 is a shorter form to produce [1 1 1 0 0 ··· 0] (n−1 terms).



                                                                                                            filter(1,[1 0 -1 -1],[1 1 1 0 0 ··· 0]) passes the input [1 1 1 0 0 ··· 0] through a discrete-time filter defined by a transfer function with numerator coefficient 1 and denominator coefficients [1 0 -1 -1].






                                                                                                            share|improve this answer











                                                                                                            $endgroup$




                                                                                                            Octave / MATLAB, 35 33 bytes





                                                                                                            @(n)[1 filter(1,'cbaa'-98,2:n<5)]


                                                                                                            Outputs the first n terms.



                                                                                                            Try it online!



                                                                                                            How it works



                                                                                                            Anonymous function that implements a recursive filter.



                                                                                                            'cbaa'-98 is a shorter form to produce [1 0 -1 -1].



                                                                                                            2:n<5 is a shorter form to produce [1 1 1 0 0 ··· 0] (n−1 terms).



                                                                                                            filter(1,[1 0 -1 -1],[1 1 1 0 0 ··· 0]) passes the input [1 1 1 0 0 ··· 0] through a discrete-time filter defined by a transfer function with numerator coefficient 1 and denominator coefficients [1 0 -1 -1].







                                                                                                            share|improve this answer














                                                                                                            share|improve this answer



                                                                                                            share|improve this answer








                                                                                                            edited 19 hours ago

























                                                                                                            answered 19 hours ago









                                                                                                            Luis MendoLuis Mendo

                                                                                                            75.3k889292




                                                                                                            75.3k889292























                                                                                                                3












                                                                                                                $begingroup$


                                                                                                                Jelly, 11 bytes



                                                                                                                5B+Ɲ2ị;Ʋ⁸¡Ḣ


                                                                                                                Try it online!



                                                                                                                0-indexed.






                                                                                                                share|improve this answer











                                                                                                                $endgroup$













                                                                                                                • $begingroup$
                                                                                                                  Can you specify whether this answer is 0-indexed or 1-indexed?
                                                                                                                  $endgroup$
                                                                                                                  – Tau
                                                                                                                  yesterday










                                                                                                                • $begingroup$
                                                                                                                  @Tau It's 0-indexed. I've edited it in.
                                                                                                                  $endgroup$
                                                                                                                  – Erik the Outgolfer
                                                                                                                  yesterday


















                                                                                                                3












                                                                                                                $begingroup$


                                                                                                                Jelly, 11 bytes



                                                                                                                5B+Ɲ2ị;Ʋ⁸¡Ḣ


                                                                                                                Try it online!



                                                                                                                0-indexed.






                                                                                                                share|improve this answer











                                                                                                                $endgroup$













                                                                                                                • $begingroup$
                                                                                                                  Can you specify whether this answer is 0-indexed or 1-indexed?
                                                                                                                  $endgroup$
                                                                                                                  – Tau
                                                                                                                  yesterday










                                                                                                                • $begingroup$
                                                                                                                  @Tau It's 0-indexed. I've edited it in.
                                                                                                                  $endgroup$
                                                                                                                  – Erik the Outgolfer
                                                                                                                  yesterday
















                                                                                                                3












                                                                                                                3








                                                                                                                3





                                                                                                                $begingroup$


                                                                                                                Jelly, 11 bytes



                                                                                                                5B+Ɲ2ị;Ʋ⁸¡Ḣ


                                                                                                                Try it online!



                                                                                                                0-indexed.






                                                                                                                share|improve this answer











                                                                                                                $endgroup$




                                                                                                                Jelly, 11 bytes



                                                                                                                5B+Ɲ2ị;Ʋ⁸¡Ḣ


                                                                                                                Try it online!



                                                                                                                0-indexed.







                                                                                                                share|improve this answer














                                                                                                                share|improve this answer



                                                                                                                share|improve this answer








                                                                                                                edited yesterday

























                                                                                                                answered yesterday









                                                                                                                Erik the OutgolferErik the Outgolfer

                                                                                                                33k429106




                                                                                                                33k429106












                                                                                                                • $begingroup$
                                                                                                                  Can you specify whether this answer is 0-indexed or 1-indexed?
                                                                                                                  $endgroup$
                                                                                                                  – Tau
                                                                                                                  yesterday










                                                                                                                • $begingroup$
                                                                                                                  @Tau It's 0-indexed. I've edited it in.
                                                                                                                  $endgroup$
                                                                                                                  – Erik the Outgolfer
                                                                                                                  yesterday




















                                                                                                                • $begingroup$
                                                                                                                  Can you specify whether this answer is 0-indexed or 1-indexed?
                                                                                                                  $endgroup$
                                                                                                                  – Tau
                                                                                                                  yesterday










                                                                                                                • $begingroup$
                                                                                                                  @Tau It's 0-indexed. I've edited it in.
                                                                                                                  $endgroup$
                                                                                                                  – Erik the Outgolfer
                                                                                                                  yesterday


















                                                                                                                $begingroup$
                                                                                                                Can you specify whether this answer is 0-indexed or 1-indexed?
                                                                                                                $endgroup$
                                                                                                                – Tau
                                                                                                                yesterday




                                                                                                                $begingroup$
                                                                                                                Can you specify whether this answer is 0-indexed or 1-indexed?
                                                                                                                $endgroup$
                                                                                                                – Tau
                                                                                                                yesterday












                                                                                                                $begingroup$
                                                                                                                @Tau It's 0-indexed. I've edited it in.
                                                                                                                $endgroup$
                                                                                                                – Erik the Outgolfer
                                                                                                                yesterday






                                                                                                                $begingroup$
                                                                                                                @Tau It's 0-indexed. I've edited it in.
                                                                                                                $endgroup$
                                                                                                                – Erik the Outgolfer
                                                                                                                yesterday













                                                                                                                3












                                                                                                                $begingroup$


                                                                                                                Perl 6, 24 bytes



                                                                                                                {(1,1,1,*+*+!*...*)[$_]}


                                                                                                                Try it online!



                                                                                                                A pretty standard generated sequence, with each new element generated by the expression * + * + !*. That adds the third-previous element, the second-previous element, and the logical negation of the previous element, which is always False, which is numerically zero.






                                                                                                                share|improve this answer











                                                                                                                $endgroup$













                                                                                                                • $begingroup$
                                                                                                                  Why is this community wiki?
                                                                                                                  $endgroup$
                                                                                                                  – Jo King
                                                                                                                  yesterday










                                                                                                                • $begingroup$
                                                                                                                  @JoKing Beats me. If I did it somehow, it wasn't on purpose.
                                                                                                                  $endgroup$
                                                                                                                  – Sean
                                                                                                                  12 hours ago
















                                                                                                                3












                                                                                                                $begingroup$


                                                                                                                Perl 6, 24 bytes



                                                                                                                {(1,1,1,*+*+!*...*)[$_]}


                                                                                                                Try it online!



                                                                                                                A pretty standard generated sequence, with each new element generated by the expression * + * + !*. That adds the third-previous element, the second-previous element, and the logical negation of the previous element, which is always False, which is numerically zero.






                                                                                                                share|improve this answer











                                                                                                                $endgroup$













                                                                                                                • $begingroup$
                                                                                                                  Why is this community wiki?
                                                                                                                  $endgroup$
                                                                                                                  – Jo King
                                                                                                                  yesterday










                                                                                                                • $begingroup$
                                                                                                                  @JoKing Beats me. If I did it somehow, it wasn't on purpose.
                                                                                                                  $endgroup$
                                                                                                                  – Sean
                                                                                                                  12 hours ago














                                                                                                                3












                                                                                                                3








                                                                                                                3





                                                                                                                $begingroup$


                                                                                                                Perl 6, 24 bytes



                                                                                                                {(1,1,1,*+*+!*...*)[$_]}


                                                                                                                Try it online!



                                                                                                                A pretty standard generated sequence, with each new element generated by the expression * + * + !*. That adds the third-previous element, the second-previous element, and the logical negation of the previous element, which is always False, which is numerically zero.






                                                                                                                share|improve this answer











                                                                                                                $endgroup$




                                                                                                                Perl 6, 24 bytes



                                                                                                                {(1,1,1,*+*+!*...*)[$_]}


                                                                                                                Try it online!



                                                                                                                A pretty standard generated sequence, with each new element generated by the expression * + * + !*. That adds the third-previous element, the second-previous element, and the logical negation of the previous element, which is always False, which is numerically zero.







                                                                                                                share|improve this answer














                                                                                                                share|improve this answer



                                                                                                                share|improve this answer








                                                                                                                answered yesterday


























                                                                                                                community wiki





                                                                                                                Sean













                                                                                                                • $begingroup$
                                                                                                                  Why is this community wiki?
                                                                                                                  $endgroup$
                                                                                                                  – Jo King
                                                                                                                  yesterday










                                                                                                                • $begingroup$
                                                                                                                  @JoKing Beats me. If I did it somehow, it wasn't on purpose.
                                                                                                                  $endgroup$
                                                                                                                  – Sean
                                                                                                                  12 hours ago


















                                                                                                                • $begingroup$
                                                                                                                  Why is this community wiki?
                                                                                                                  $endgroup$
                                                                                                                  – Jo King
                                                                                                                  yesterday










                                                                                                                • $begingroup$
                                                                                                                  @JoKing Beats me. If I did it somehow, it wasn't on purpose.
                                                                                                                  $endgroup$
                                                                                                                  – Sean
                                                                                                                  12 hours ago
















                                                                                                                $begingroup$
                                                                                                                Why is this community wiki?
                                                                                                                $endgroup$
                                                                                                                – Jo King
                                                                                                                yesterday




                                                                                                                $begingroup$
                                                                                                                Why is this community wiki?
                                                                                                                $endgroup$
                                                                                                                – Jo King
                                                                                                                yesterday












                                                                                                                $begingroup$
                                                                                                                @JoKing Beats me. If I did it somehow, it wasn't on purpose.
                                                                                                                $endgroup$
                                                                                                                – Sean
                                                                                                                12 hours ago




                                                                                                                $begingroup$
                                                                                                                @JoKing Beats me. If I did it somehow, it wasn't on purpose.
                                                                                                                $endgroup$
                                                                                                                – Sean
                                                                                                                12 hours ago











                                                                                                                3












                                                                                                                $begingroup$


                                                                                                                Lua 5.3, 49 48 bytes





                                                                                                                function f(n)return n<4 and 1or f(n-2)+f(n-3)end


                                                                                                                Try it online!



                                                                                                                Vanilla Lua doesn't have coercion of booleans to strings (even tonumber(true) returns nil), so you have to use a pseudo-ternary operator. This version is 1-indexed, like all of Lua. The 1or part has to be changed to 1 or in Lua 5.1, which has a different way of lexing numbers.






                                                                                                                share|improve this answer











                                                                                                                $endgroup$


















                                                                                                                  3












                                                                                                                  $begingroup$


                                                                                                                  Lua 5.3, 49 48 bytes





                                                                                                                  function f(n)return n<4 and 1or f(n-2)+f(n-3)end


                                                                                                                  Try it online!



                                                                                                                  Vanilla Lua doesn't have coercion of booleans to strings (even tonumber(true) returns nil), so you have to use a pseudo-ternary operator. This version is 1-indexed, like all of Lua. The 1or part has to be changed to 1 or in Lua 5.1, which has a different way of lexing numbers.






                                                                                                                  share|improve this answer











                                                                                                                  $endgroup$
















                                                                                                                    3












                                                                                                                    3








                                                                                                                    3





                                                                                                                    $begingroup$


                                                                                                                    Lua 5.3, 49 48 bytes





                                                                                                                    function f(n)return n<4 and 1or f(n-2)+f(n-3)end


                                                                                                                    Try it online!



                                                                                                                    Vanilla Lua doesn't have coercion of booleans to strings (even tonumber(true) returns nil), so you have to use a pseudo-ternary operator. This version is 1-indexed, like all of Lua. The 1or part has to be changed to 1 or in Lua 5.1, which has a different way of lexing numbers.






                                                                                                                    share|improve this answer











                                                                                                                    $endgroup$




                                                                                                                    Lua 5.3, 49 48 bytes





                                                                                                                    function f(n)return n<4 and 1or f(n-2)+f(n-3)end


                                                                                                                    Try it online!



                                                                                                                    Vanilla Lua doesn't have coercion of booleans to strings (even tonumber(true) returns nil), so you have to use a pseudo-ternary operator. This version is 1-indexed, like all of Lua. The 1or part has to be changed to 1 or in Lua 5.1, which has a different way of lexing numbers.







                                                                                                                    share|improve this answer














                                                                                                                    share|improve this answer



                                                                                                                    share|improve this answer








                                                                                                                    edited 22 hours ago

























                                                                                                                    answered yesterday









                                                                                                                    cyclaministcyclaminist

                                                                                                                    1813




                                                                                                                    1813























                                                                                                                        3












                                                                                                                        $begingroup$


                                                                                                                        APL (Dyalog Unicode), 20 18 17 bytesSBCS



                                                                                                                        This code is 1-indexed. It's the same number of bytes to get n items of the Padovan sequence, as you have to drop the last few extra members. It's also the same number of bytes to get 0-indexing.



                                                                                                                        Edit: -2 bytes thanks to ngn. -1 byte thanks to ngn



                                                                                                                        4⌷2(⊢,⍨2⌷+/)⍣⎕×⍳3


                                                                                                                        Try it online!



                                                                                                                        Explanation



                                                                                                                        4⌷2(⊢,⍨2⌷+/)⍣⎕×⍳3

                                                                                                                        ⍺(. . . .)⍣⎕⍵ This format simply takes the input ⎕ and applies the function
                                                                                                                        inside the brackets (...) to its operands (here marked ⍵ and ⍺).
                                                                                                                        2(. . .+/)⍣⎕×⍳3 In this case, our ⍵, the left argument, is the array 1 1 1,
                                                                                                                        where we save our results as the function is repeatedly applied
                                                                                                                        and our ⍺, 2, is our right argument and is immediately applied to +/,
                                                                                                                        so that we have 2+/ which will return the pairwise sums of our array.
                                                                                                                        2⌷ We take the second pairwise sum, f(n-2) + f(n-3)
                                                                                                                        ⊢,⍨ And add it to the head of our array.
                                                                                                                        4⌷ When we've finished adding Padovan numbers to the end of our list,
                                                                                                                        the n-th Padovan number (1-indexed) is the 4th member of that list,
                                                                                                                        and so, we implicitly return that.





                                                                                                                        share|improve this answer











                                                                                                                        $endgroup$


















                                                                                                                          3












                                                                                                                          $begingroup$


                                                                                                                          APL (Dyalog Unicode), 20 18 17 bytesSBCS



                                                                                                                          This code is 1-indexed. It's the same number of bytes to get n items of the Padovan sequence, as you have to drop the last few extra members. It's also the same number of bytes to get 0-indexing.



                                                                                                                          Edit: -2 bytes thanks to ngn. -1 byte thanks to ngn



                                                                                                                          4⌷2(⊢,⍨2⌷+/)⍣⎕×⍳3


                                                                                                                          Try it online!



                                                                                                                          Explanation



                                                                                                                          4⌷2(⊢,⍨2⌷+/)⍣⎕×⍳3

                                                                                                                          ⍺(. . . .)⍣⎕⍵ This format simply takes the input ⎕ and applies the function
                                                                                                                          inside the brackets (...) to its operands (here marked ⍵ and ⍺).
                                                                                                                          2(. . .+/)⍣⎕×⍳3 In this case, our ⍵, the left argument, is the array 1 1 1,
                                                                                                                          where we save our results as the function is repeatedly applied
                                                                                                                          and our ⍺, 2, is our right argument and is immediately applied to +/,
                                                                                                                          so that we have 2+/ which will return the pairwise sums of our array.
                                                                                                                          2⌷ We take the second pairwise sum, f(n-2) + f(n-3)
                                                                                                                          ⊢,⍨ And add it to the head of our array.
                                                                                                                          4⌷ When we've finished adding Padovan numbers to the end of our list,
                                                                                                                          the n-th Padovan number (1-indexed) is the 4th member of that list,
                                                                                                                          and so, we implicitly return that.





                                                                                                                          share|improve this answer











                                                                                                                          $endgroup$
















                                                                                                                            3












                                                                                                                            3








                                                                                                                            3





                                                                                                                            $begingroup$


                                                                                                                            APL (Dyalog Unicode), 20 18 17 bytesSBCS



                                                                                                                            This code is 1-indexed. It's the same number of bytes to get n items of the Padovan sequence, as you have to drop the last few extra members. It's also the same number of bytes to get 0-indexing.



                                                                                                                            Edit: -2 bytes thanks to ngn. -1 byte thanks to ngn



                                                                                                                            4⌷2(⊢,⍨2⌷+/)⍣⎕×⍳3


                                                                                                                            Try it online!



                                                                                                                            Explanation



                                                                                                                            4⌷2(⊢,⍨2⌷+/)⍣⎕×⍳3

                                                                                                                            ⍺(. . . .)⍣⎕⍵ This format simply takes the input ⎕ and applies the function
                                                                                                                            inside the brackets (...) to its operands (here marked ⍵ and ⍺).
                                                                                                                            2(. . .+/)⍣⎕×⍳3 In this case, our ⍵, the left argument, is the array 1 1 1,
                                                                                                                            where we save our results as the function is repeatedly applied
                                                                                                                            and our ⍺, 2, is our right argument and is immediately applied to +/,
                                                                                                                            so that we have 2+/ which will return the pairwise sums of our array.
                                                                                                                            2⌷ We take the second pairwise sum, f(n-2) + f(n-3)
                                                                                                                            ⊢,⍨ And add it to the head of our array.
                                                                                                                            4⌷ When we've finished adding Padovan numbers to the end of our list,
                                                                                                                            the n-th Padovan number (1-indexed) is the 4th member of that list,
                                                                                                                            and so, we implicitly return that.





                                                                                                                            share|improve this answer











                                                                                                                            $endgroup$




                                                                                                                            APL (Dyalog Unicode), 20 18 17 bytesSBCS



                                                                                                                            This code is 1-indexed. It's the same number of bytes to get n items of the Padovan sequence, as you have to drop the last few extra members. It's also the same number of bytes to get 0-indexing.



                                                                                                                            Edit: -2 bytes thanks to ngn. -1 byte thanks to ngn



                                                                                                                            4⌷2(⊢,⍨2⌷+/)⍣⎕×⍳3


                                                                                                                            Try it online!



                                                                                                                            Explanation



                                                                                                                            4⌷2(⊢,⍨2⌷+/)⍣⎕×⍳3

                                                                                                                            ⍺(. . . .)⍣⎕⍵ This format simply takes the input ⎕ and applies the function
                                                                                                                            inside the brackets (...) to its operands (here marked ⍵ and ⍺).
                                                                                                                            2(. . .+/)⍣⎕×⍳3 In this case, our ⍵, the left argument, is the array 1 1 1,
                                                                                                                            where we save our results as the function is repeatedly applied
                                                                                                                            and our ⍺, 2, is our right argument and is immediately applied to +/,
                                                                                                                            so that we have 2+/ which will return the pairwise sums of our array.
                                                                                                                            2⌷ We take the second pairwise sum, f(n-2) + f(n-3)
                                                                                                                            ⊢,⍨ And add it to the head of our array.
                                                                                                                            4⌷ When we've finished adding Padovan numbers to the end of our list,
                                                                                                                            the n-th Padovan number (1-indexed) is the 4th member of that list,
                                                                                                                            and so, we implicitly return that.






                                                                                                                            share|improve this answer














                                                                                                                            share|improve this answer



                                                                                                                            share|improve this answer








                                                                                                                            edited 15 hours ago

























                                                                                                                            answered 19 hours ago









                                                                                                                            Sherlock9Sherlock9

                                                                                                                            8,16411860




                                                                                                                            8,16411860























                                                                                                                                3












                                                                                                                                $begingroup$


                                                                                                                                K (ngn/k), 24 20 bytes



                                                                                                                                -4 bytes thanks to ngn!



                                                                                                                                {$[x<3;1;+/o'x-2 3]}


                                                                                                                                Try it online!



                                                                                                                                0-indexed, first N terms






                                                                                                                                share|improve this answer











                                                                                                                                $endgroup$









                                                                                                                                • 1




                                                                                                                                  $begingroup$
                                                                                                                                  f[x-2]+f[x-3] -> +/o'x-2 3 (o is "recur")
                                                                                                                                  $endgroup$
                                                                                                                                  – ngn
                                                                                                                                  11 hours ago










                                                                                                                                • $begingroup$
                                                                                                                                  @ngn Thanks! I tried it (without success) in J; it's elegant here.
                                                                                                                                  $endgroup$
                                                                                                                                  – Galen Ivanov
                                                                                                                                  10 hours ago










                                                                                                                                • $begingroup$
                                                                                                                                  @ngn In fact here's one possibillity how it looks in J: (1#.2 3$:@-~])`1:@.(3&>)
                                                                                                                                  $endgroup$
                                                                                                                                  – Galen Ivanov
                                                                                                                                  9 hours ago










                                                                                                                                • $begingroup$
                                                                                                                                  ah, right, base-1 decode is a train-friendly way to sum :)
                                                                                                                                  $endgroup$
                                                                                                                                  – ngn
                                                                                                                                  9 hours ago






                                                                                                                                • 2




                                                                                                                                  $begingroup$
                                                                                                                                  1: -> # in the j solution
                                                                                                                                  $endgroup$
                                                                                                                                  – ngn
                                                                                                                                  9 hours ago


















                                                                                                                                3












                                                                                                                                $begingroup$


                                                                                                                                K (ngn/k), 24 20 bytes



                                                                                                                                -4 bytes thanks to ngn!



                                                                                                                                {$[x<3;1;+/o'x-2 3]}


                                                                                                                                Try it online!



                                                                                                                                0-indexed, first N terms






                                                                                                                                share|improve this answer











                                                                                                                                $endgroup$









                                                                                                                                • 1




                                                                                                                                  $begingroup$
                                                                                                                                  f[x-2]+f[x-3] -> +/o'x-2 3 (o is "recur")
                                                                                                                                  $endgroup$
                                                                                                                                  – ngn
                                                                                                                                  11 hours ago










                                                                                                                                • $begingroup$
                                                                                                                                  @ngn Thanks! I tried it (without success) in J; it's elegant here.
                                                                                                                                  $endgroup$
                                                                                                                                  – Galen Ivanov
                                                                                                                                  10 hours ago










                                                                                                                                • $begingroup$
                                                                                                                                  @ngn In fact here's one possibillity how it looks in J: (1#.2 3$:@-~])`1:@.(3&>)
                                                                                                                                  $endgroup$
                                                                                                                                  – Galen Ivanov
                                                                                                                                  9 hours ago










                                                                                                                                • $begingroup$
                                                                                                                                  ah, right, base-1 decode is a train-friendly way to sum :)
                                                                                                                                  $endgroup$
                                                                                                                                  – ngn
                                                                                                                                  9 hours ago






                                                                                                                                • 2




                                                                                                                                  $begingroup$
                                                                                                                                  1: -> # in the j solution
                                                                                                                                  $endgroup$
                                                                                                                                  – ngn
                                                                                                                                  9 hours ago
















                                                                                                                                3












                                                                                                                                3








                                                                                                                                3





                                                                                                                                $begingroup$


                                                                                                                                K (ngn/k), 24 20 bytes



                                                                                                                                -4 bytes thanks to ngn!



                                                                                                                                {$[x<3;1;+/o'x-2 3]}


                                                                                                                                Try it online!



                                                                                                                                0-indexed, first N terms






                                                                                                                                share|improve this answer











                                                                                                                                $endgroup$




                                                                                                                                K (ngn/k), 24 20 bytes



                                                                                                                                -4 bytes thanks to ngn!



                                                                                                                                {$[x<3;1;+/o'x-2 3]}


                                                                                                                                Try it online!



                                                                                                                                0-indexed, first N terms







                                                                                                                                share|improve this answer














                                                                                                                                share|improve this answer



                                                                                                                                share|improve this answer








                                                                                                                                edited 9 hours ago

























                                                                                                                                answered 21 hours ago









                                                                                                                                Galen IvanovGalen Ivanov

                                                                                                                                7,41211034




                                                                                                                                7,41211034








                                                                                                                                • 1




                                                                                                                                  $begingroup$
                                                                                                                                  f[x-2]+f[x-3] -> +/o'x-2 3 (o is "recur")
                                                                                                                                  $endgroup$
                                                                                                                                  – ngn
                                                                                                                                  11 hours ago










                                                                                                                                • $begingroup$
                                                                                                                                  @ngn Thanks! I tried it (without success) in J; it's elegant here.
                                                                                                                                  $endgroup$
                                                                                                                                  – Galen Ivanov
                                                                                                                                  10 hours ago










                                                                                                                                • $begingroup$
                                                                                                                                  @ngn In fact here's one possibillity how it looks in J: (1#.2 3$:@-~])`1:@.(3&>)
                                                                                                                                  $endgroup$
                                                                                                                                  – Galen Ivanov
                                                                                                                                  9 hours ago










                                                                                                                                • $begingroup$
                                                                                                                                  ah, right, base-1 decode is a train-friendly way to sum :)
                                                                                                                                  $endgroup$
                                                                                                                                  – ngn
                                                                                                                                  9 hours ago






                                                                                                                                • 2




                                                                                                                                  $begingroup$
                                                                                                                                  1: -> # in the j solution
                                                                                                                                  $endgroup$
                                                                                                                                  – ngn
                                                                                                                                  9 hours ago
















                                                                                                                                • 1




                                                                                                                                  $begingroup$
                                                                                                                                  f[x-2]+f[x-3] -> +/o'x-2 3 (o is "recur")
                                                                                                                                  $endgroup$
                                                                                                                                  – ngn
                                                                                                                                  11 hours ago










                                                                                                                                • $begingroup$
                                                                                                                                  @ngn Thanks! I tried it (without success) in J; it's elegant here.
                                                                                                                                  $endgroup$
                                                                                                                                  – Galen Ivanov
                                                                                                                                  10 hours ago










                                                                                                                                • $begingroup$
                                                                                                                                  @ngn In fact here's one possibillity how it looks in J: (1#.2 3$:@-~])`1:@.(3&>)
                                                                                                                                  $endgroup$
                                                                                                                                  – Galen Ivanov
                                                                                                                                  9 hours ago










                                                                                                                                • $begingroup$
                                                                                                                                  ah, right, base-1 decode is a train-friendly way to sum :)
                                                                                                                                  $endgroup$
                                                                                                                                  – ngn
                                                                                                                                  9 hours ago






                                                                                                                                • 2




                                                                                                                                  $begingroup$
                                                                                                                                  1: -> # in the j solution
                                                                                                                                  $endgroup$
                                                                                                                                  – ngn
                                                                                                                                  9 hours ago










                                                                                                                                1




                                                                                                                                1




                                                                                                                                $begingroup$
                                                                                                                                f[x-2]+f[x-3] -> +/o'x-2 3 (o is "recur")
                                                                                                                                $endgroup$
                                                                                                                                – ngn
                                                                                                                                11 hours ago




                                                                                                                                $begingroup$
                                                                                                                                f[x-2]+f[x-3] -> +/o'x-2 3 (o is "recur")
                                                                                                                                $endgroup$
                                                                                                                                – ngn
                                                                                                                                11 hours ago












                                                                                                                                $begingroup$
                                                                                                                                @ngn Thanks! I tried it (without success) in J; it's elegant here.
                                                                                                                                $endgroup$
                                                                                                                                – Galen Ivanov
                                                                                                                                10 hours ago




                                                                                                                                $begingroup$
                                                                                                                                @ngn Thanks! I tried it (without success) in J; it's elegant here.
                                                                                                                                $endgroup$
                                                                                                                                – Galen Ivanov
                                                                                                                                10 hours ago












                                                                                                                                $begingroup$
                                                                                                                                @ngn In fact here's one possibillity how it looks in J: (1#.2 3$:@-~])`1:@.(3&>)
                                                                                                                                $endgroup$
                                                                                                                                – Galen Ivanov
                                                                                                                                9 hours ago




                                                                                                                                $begingroup$
                                                                                                                                @ngn In fact here's one possibillity how it looks in J: (1#.2 3$:@-~])`1:@.(3&>)
                                                                                                                                $endgroup$
                                                                                                                                – Galen Ivanov
                                                                                                                                9 hours ago












                                                                                                                                $begingroup$
                                                                                                                                ah, right, base-1 decode is a train-friendly way to sum :)
                                                                                                                                $endgroup$
                                                                                                                                – ngn
                                                                                                                                9 hours ago




                                                                                                                                $begingroup$
                                                                                                                                ah, right, base-1 decode is a train-friendly way to sum :)
                                                                                                                                $endgroup$
                                                                                                                                – ngn
                                                                                                                                9 hours ago




                                                                                                                                2




                                                                                                                                2




                                                                                                                                $begingroup$
                                                                                                                                1: -> # in the j solution
                                                                                                                                $endgroup$
                                                                                                                                – ngn
                                                                                                                                9 hours ago






                                                                                                                                $begingroup$
                                                                                                                                1: -> # in the j solution
                                                                                                                                $endgroup$
                                                                                                                                – ngn
                                                                                                                                9 hours ago













                                                                                                                                2












                                                                                                                                $begingroup$

                                                                                                                                JavaScript (ES6), 23 bytes



                                                                                                                                Implements the recursive definition of A000931. Returns the $N$th term, 0-indexed.





                                                                                                                                f=n=>n<3||f(n-2)+f(n-3)


                                                                                                                                Try it online!






                                                                                                                                share|improve this answer











                                                                                                                                $endgroup$













                                                                                                                                • $begingroup$
                                                                                                                                  I don't think it's reasonable to say that returning true is the same as returning 1 if the rest of the output is numbers.
                                                                                                                                  $endgroup$
                                                                                                                                  – Nit
                                                                                                                                  17 hours ago










                                                                                                                                • $begingroup$
                                                                                                                                  @Nit Relevant meta post.
                                                                                                                                  $endgroup$
                                                                                                                                  – Arnauld
                                                                                                                                  16 hours ago
















                                                                                                                                2












                                                                                                                                $begingroup$

                                                                                                                                JavaScript (ES6), 23 bytes



                                                                                                                                Implements the recursive definition of A000931. Returns the $N$th term, 0-indexed.





                                                                                                                                f=n=>n<3||f(n-2)+f(n-3)


                                                                                                                                Try it online!






                                                                                                                                share|improve this answer











                                                                                                                                $endgroup$













                                                                                                                                • $begingroup$
                                                                                                                                  I don't think it's reasonable to say that returning true is the same as returning 1 if the rest of the output is numbers.
                                                                                                                                  $endgroup$
                                                                                                                                  – Nit
                                                                                                                                  17 hours ago










                                                                                                                                • $begingroup$
                                                                                                                                  @Nit Relevant meta post.
                                                                                                                                  $endgroup$
                                                                                                                                  – Arnauld
                                                                                                                                  16 hours ago














                                                                                                                                2












                                                                                                                                2








                                                                                                                                2





                                                                                                                                $begingroup$

                                                                                                                                JavaScript (ES6), 23 bytes



                                                                                                                                Implements the recursive definition of A000931. Returns the $N$th term, 0-indexed.





                                                                                                                                f=n=>n<3||f(n-2)+f(n-3)


                                                                                                                                Try it online!






                                                                                                                                share|improve this answer











                                                                                                                                $endgroup$



                                                                                                                                JavaScript (ES6), 23 bytes



                                                                                                                                Implements the recursive definition of A000931. Returns the $N$th term, 0-indexed.





                                                                                                                                f=n=>n<3||f(n-2)+f(n-3)


                                                                                                                                Try it online!







                                                                                                                                share|improve this answer














                                                                                                                                share|improve this answer



                                                                                                                                share|improve this answer








                                                                                                                                edited yesterday

























                                                                                                                                answered yesterday









                                                                                                                                ArnauldArnauld

                                                                                                                                80.6k797334




                                                                                                                                80.6k797334












                                                                                                                                • $begingroup$
                                                                                                                                  I don't think it's reasonable to say that returning true is the same as returning 1 if the rest of the output is numbers.
                                                                                                                                  $endgroup$
                                                                                                                                  – Nit
                                                                                                                                  17 hours ago










                                                                                                                                • $begingroup$
                                                                                                                                  @Nit Relevant meta post.
                                                                                                                                  $endgroup$
                                                                                                                                  – Arnauld
                                                                                                                                  16 hours ago


















                                                                                                                                • $begingroup$
                                                                                                                                  I don't think it's reasonable to say that returning true is the same as returning 1 if the rest of the output is numbers.
                                                                                                                                  $endgroup$
                                                                                                                                  – Nit
                                                                                                                                  17 hours ago










                                                                                                                                • $begingroup$
                                                                                                                                  @Nit Relevant meta post.
                                                                                                                                  $endgroup$
                                                                                                                                  – Arnauld
                                                                                                                                  16 hours ago
















                                                                                                                                $begingroup$
                                                                                                                                I don't think it's reasonable to say that returning true is the same as returning 1 if the rest of the output is numbers.
                                                                                                                                $endgroup$
                                                                                                                                – Nit
                                                                                                                                17 hours ago




                                                                                                                                $begingroup$
                                                                                                                                I don't think it's reasonable to say that returning true is the same as returning 1 if the rest of the output is numbers.
                                                                                                                                $endgroup$
                                                                                                                                – Nit
                                                                                                                                17 hours ago












                                                                                                                                $begingroup$
                                                                                                                                @Nit Relevant meta post.
                                                                                                                                $endgroup$
                                                                                                                                – Arnauld
                                                                                                                                16 hours ago




                                                                                                                                $begingroup$
                                                                                                                                @Nit Relevant meta post.
                                                                                                                                $endgroup$
                                                                                                                                – Arnauld
                                                                                                                                16 hours ago











                                                                                                                                2












                                                                                                                                $begingroup$


                                                                                                                                Japt -N, 12 bytes



                                                                                                                                <3ªßUµ2 +ß´U


                                                                                                                                Try it






                                                                                                                                share|improve this answer









                                                                                                                                $endgroup$













                                                                                                                                • $begingroup$
                                                                                                                                  Looks like 12 is the best we can do :
                                                                                                                                  $endgroup$
                                                                                                                                  – Shaggy
                                                                                                                                  yesterday
















                                                                                                                                2












                                                                                                                                $begingroup$


                                                                                                                                Japt -N, 12 bytes



                                                                                                                                <3ªßUµ2 +ß´U


                                                                                                                                Try it






                                                                                                                                share|improve this answer









                                                                                                                                $endgroup$













                                                                                                                                • $begingroup$
                                                                                                                                  Looks like 12 is the best we can do :
                                                                                                                                  $endgroup$
                                                                                                                                  – Shaggy
                                                                                                                                  yesterday














                                                                                                                                2












                                                                                                                                2








                                                                                                                                2





                                                                                                                                $begingroup$


                                                                                                                                Japt -N, 12 bytes



                                                                                                                                <3ªßUµ2 +ß´U


                                                                                                                                Try it






                                                                                                                                share|improve this answer









                                                                                                                                $endgroup$




                                                                                                                                Japt -N, 12 bytes



                                                                                                                                <3ªßUµ2 +ß´U


                                                                                                                                Try it







                                                                                                                                share|improve this answer












                                                                                                                                share|improve this answer



                                                                                                                                share|improve this answer










                                                                                                                                answered yesterday









                                                                                                                                Embodiment of IgnoranceEmbodiment of Ignorance

                                                                                                                                2,866127




                                                                                                                                2,866127












                                                                                                                                • $begingroup$
                                                                                                                                  Looks like 12 is the best we can do :
                                                                                                                                  $endgroup$
                                                                                                                                  – Shaggy
                                                                                                                                  yesterday


















                                                                                                                                • $begingroup$
                                                                                                                                  Looks like 12 is the best we can do :
                                                                                                                                  $endgroup$
                                                                                                                                  – Shaggy
                                                                                                                                  yesterday
















                                                                                                                                $begingroup$
                                                                                                                                Looks like 12 is the best we can do :
                                                                                                                                $endgroup$
                                                                                                                                – Shaggy
                                                                                                                                yesterday




                                                                                                                                $begingroup$
                                                                                                                                Looks like 12 is the best we can do :
                                                                                                                                $endgroup$
                                                                                                                                – Shaggy
                                                                                                                                yesterday











                                                                                                                                2












                                                                                                                                $begingroup$

                                                                                                                                Pyth, 16 bytes



                                                                                                                                L?<b3!b+y-b2y-b3


                                                                                                                                This defines the function y. Try it here!



                                                                                                                                Here's a more fun solution, though it's 9 bytes longer; bytes could be shaved though.



                                                                                                                                +l{sa.pMf.Am&>d2%d2T./QY!


                                                                                                                                This uses the definition given by David Callan on the OEIS page: "a(n) = number of compositions of n into parts that are odd and >= 3." Try it here! It takes input directly instead of defining a function.






                                                                                                                                share|improve this answer









                                                                                                                                $endgroup$













                                                                                                                                • $begingroup$
                                                                                                                                  y-b2y-b3 could maybe be refactored with either bifurcate or L? Though declaring an array of 2 elements is costly. yL-Lb2,3 is longer :(
                                                                                                                                  $endgroup$
                                                                                                                                  – Ven
                                                                                                                                  21 hours ago












                                                                                                                                • $begingroup$
                                                                                                                                  @Ven I was able to replace +y-b2y-b3 with smy-bdhB2 which is the same amount of bytes; hB2 results in the array [2, 3]
                                                                                                                                  $endgroup$
                                                                                                                                  – RK.
                                                                                                                                  13 hours ago










                                                                                                                                • $begingroup$
                                                                                                                                  Well done on hB2. Too bad it's the same byte count.
                                                                                                                                  $endgroup$
                                                                                                                                  – Ven
                                                                                                                                  13 hours ago










                                                                                                                                • $begingroup$
                                                                                                                                  Yeah, though I wonder if there's some way to get rid of the d in the map.
                                                                                                                                  $endgroup$
                                                                                                                                  – RK.
                                                                                                                                  13 hours ago
















                                                                                                                                2












                                                                                                                                $begingroup$

                                                                                                                                Pyth, 16 bytes



                                                                                                                                L?<b3!b+y-b2y-b3


                                                                                                                                This defines the function y. Try it here!



                                                                                                                                Here's a more fun solution, though it's 9 bytes longer; bytes could be shaved though.



                                                                                                                                +l{sa.pMf.Am&>d2%d2T./QY!


                                                                                                                                This uses the definition given by David Callan on the OEIS page: "a(n) = number of compositions of n into parts that are odd and >= 3." Try it here! It takes input directly instead of defining a function.






                                                                                                                                share|improve this answer









                                                                                                                                $endgroup$













                                                                                                                                • $begingroup$
                                                                                                                                  y-b2y-b3 could maybe be refactored with either bifurcate or L? Though declaring an array of 2 elements is costly. yL-Lb2,3 is longer :(
                                                                                                                                  $endgroup$
                                                                                                                                  – Ven
                                                                                                                                  21 hours ago












                                                                                                                                • $begingroup$
                                                                                                                                  @Ven I was able to replace +y-b2y-b3 with smy-bdhB2 which is the same amount of bytes; hB2 results in the array [2, 3]
                                                                                                                                  $endgroup$
                                                                                                                                  – RK.
                                                                                                                                  13 hours ago










                                                                                                                                • $begingroup$
                                                                                                                                  Well done on hB2. Too bad it's the same byte count.
                                                                                                                                  $endgroup$
                                                                                                                                  – Ven
                                                                                                                                  13 hours ago










                                                                                                                                • $begingroup$
                                                                                                                                  Yeah, though I wonder if there's some way to get rid of the d in the map.
                                                                                                                                  $endgroup$
                                                                                                                                  – RK.
                                                                                                                                  13 hours ago














                                                                                                                                2












                                                                                                                                2








                                                                                                                                2





                                                                                                                                $begingroup$

                                                                                                                                Pyth, 16 bytes



                                                                                                                                L?<b3!b+y-b2y-b3


                                                                                                                                This defines the function y. Try it here!



                                                                                                                                Here's a more fun solution, though it's 9 bytes longer; bytes could be shaved though.



                                                                                                                                +l{sa.pMf.Am&>d2%d2T./QY!


                                                                                                                                This uses the definition given by David Callan on the OEIS page: "a(n) = number of compositions of n into parts that are odd and >= 3." Try it here! It takes input directly instead of defining a function.






                                                                                                                                share|improve this answer









                                                                                                                                $endgroup$



                                                                                                                                Pyth, 16 bytes



                                                                                                                                L?<b3!b+y-b2y-b3


                                                                                                                                This defines the function y. Try it here!



                                                                                                                                Here's a more fun solution, though it's 9 bytes longer; bytes could be shaved though.



                                                                                                                                +l{sa.pMf.Am&>d2%d2T./QY!


                                                                                                                                This uses the definition given by David Callan on the OEIS page: "a(n) = number of compositions of n into parts that are odd and >= 3." Try it here! It takes input directly instead of defining a function.







                                                                                                                                share|improve this answer












                                                                                                                                share|improve this answer



                                                                                                                                share|improve this answer










                                                                                                                                answered yesterday









                                                                                                                                RK.RK.

                                                                                                                                407211




                                                                                                                                407211












                                                                                                                                • $begingroup$
                                                                                                                                  y-b2y-b3 could maybe be refactored with either bifurcate or L? Though declaring an array of 2 elements is costly. yL-Lb2,3 is longer :(
                                                                                                                                  $endgroup$
                                                                                                                                  – Ven
                                                                                                                                  21 hours ago












                                                                                                                                • $begingroup$
                                                                                                                                  @Ven I was able to replace +y-b2y-b3 with smy-bdhB2 which is the same amount of bytes; hB2 results in the array [2, 3]
                                                                                                                                  $endgroup$
                                                                                                                                  – RK.
                                                                                                                                  13 hours ago










                                                                                                                                • $begingroup$
                                                                                                                                  Well done on hB2. Too bad it's the same byte count.
                                                                                                                                  $endgroup$
                                                                                                                                  – Ven
                                                                                                                                  13 hours ago










                                                                                                                                • $begingroup$
                                                                                                                                  Yeah, though I wonder if there's some way to get rid of the d in the map.
                                                                                                                                  $endgroup$
                                                                                                                                  – RK.
                                                                                                                                  13 hours ago


















                                                                                                                                • $begingroup$
                                                                                                                                  y-b2y-b3 could maybe be refactored with either bifurcate or L? Though declaring an array of 2 elements is costly. yL-Lb2,3 is longer :(
                                                                                                                                  $endgroup$
                                                                                                                                  – Ven
                                                                                                                                  21 hours ago












                                                                                                                                • $begingroup$
                                                                                                                                  @Ven I was able to replace +y-b2y-b3 with smy-bdhB2 which is the same amount of bytes; hB2 results in the array [2, 3]
                                                                                                                                  $endgroup$
                                                                                                                                  – RK.
                                                                                                                                  13 hours ago










                                                                                                                                • $begingroup$
                                                                                                                                  Well done on hB2. Too bad it's the same byte count.
                                                                                                                                  $endgroup$
                                                                                                                                  – Ven
                                                                                                                                  13 hours ago










                                                                                                                                • $begingroup$
                                                                                                                                  Yeah, though I wonder if there's some way to get rid of the d in the map.
                                                                                                                                  $endgroup$
                                                                                                                                  – RK.
                                                                                                                                  13 hours ago
















                                                                                                                                $begingroup$
                                                                                                                                y-b2y-b3 could maybe be refactored with either bifurcate or L? Though declaring an array of 2 elements is costly. yL-Lb2,3 is longer :(
                                                                                                                                $endgroup$
                                                                                                                                – Ven
                                                                                                                                21 hours ago






                                                                                                                                $begingroup$
                                                                                                                                y-b2y-b3 could maybe be refactored with either bifurcate or L? Though declaring an array of 2 elements is costly. yL-Lb2,3 is longer :(
                                                                                                                                $endgroup$
                                                                                                                                – Ven
                                                                                                                                21 hours ago














                                                                                                                                $begingroup$
                                                                                                                                @Ven I was able to replace +y-b2y-b3 with smy-bdhB2 which is the same amount of bytes; hB2 results in the array [2, 3]
                                                                                                                                $endgroup$
                                                                                                                                – RK.
                                                                                                                                13 hours ago




                                                                                                                                $begingroup$
                                                                                                                                @Ven I was able to replace +y-b2y-b3 with smy-bdhB2 which is the same amount of bytes; hB2 results in the array [2, 3]
                                                                                                                                $endgroup$
                                                                                                                                – RK.
                                                                                                                                13 hours ago












                                                                                                                                $begingroup$
                                                                                                                                Well done on hB2. Too bad it's the same byte count.
                                                                                                                                $endgroup$
                                                                                                                                – Ven
                                                                                                                                13 hours ago




                                                                                                                                $begingroup$
                                                                                                                                Well done on hB2. Too bad it's the same byte count.
                                                                                                                                $endgroup$
                                                                                                                                – Ven
                                                                                                                                13 hours ago












                                                                                                                                $begingroup$
                                                                                                                                Yeah, though I wonder if there's some way to get rid of the d in the map.
                                                                                                                                $endgroup$
                                                                                                                                – RK.
                                                                                                                                13 hours ago




                                                                                                                                $begingroup$
                                                                                                                                Yeah, though I wonder if there's some way to get rid of the d in the map.
                                                                                                                                $endgroup$
                                                                                                                                – RK.
                                                                                                                                13 hours ago











                                                                                                                                2












                                                                                                                                $begingroup$


                                                                                                                                05AB1E, 8 bytes



                                                                                                                                1Ð)λ£₂₃+


                                                                                                                                Try it online!



                                                                                                                                Bear with me, I haven't golfed in a while. I wonder if there's a shorter substitute for 1Ð) which works in this case (I've tried 1D), 3Å1 etc. but none of them save bytes). Outputs the first $n$ terms of the sequence. Or, without the £, it would output an infinite stream of the terms of the sequence.



                                                                                                                                How?



                                                                                                                                1Ð)λ£₂₃+ | Full program.
                                                                                                                                1Ð) | Initialize the stack with [1, 1, 1].
                                                                                                                                λ | Begin the recursive generation of a list: Starting from some base case,
                                                                                                                                | this command generates an infinite list with the pattern function given.
                                                                                                                                £ | Flag for λ. Instead of outputting an infinite stream, only print the first n.
                                                                                                                                ₂₃+ | Add a(n-2) and a(n-3).





                                                                                                                                share|improve this answer











                                                                                                                                $endgroup$













                                                                                                                                • $begingroup$
                                                                                                                                  I don't think 1Ð) can be 2 bytes tbh. I can think of six different 3-bytes alternatives, but no 2-byters.
                                                                                                                                  $endgroup$
                                                                                                                                  – Kevin Cruijssen
                                                                                                                                  19 hours ago
















                                                                                                                                2












                                                                                                                                $begingroup$


                                                                                                                                05AB1E, 8 bytes



                                                                                                                                1Ð)λ£₂₃+


                                                                                                                                Try it online!



                                                                                                                                Bear with me, I haven't golfed in a while. I wonder if there's a shorter substitute for 1Ð) which works in this case (I've tried 1D), 3Å1 etc. but none of them save bytes). Outputs the first $n$ terms of the sequence. Or, without the £, it would output an infinite stream of the terms of the sequence.



                                                                                                                                How?



                                                                                                                                1Ð)λ£₂₃+ | Full program.
                                                                                                                                1Ð) | Initialize the stack with [1, 1, 1].
                                                                                                                                λ | Begin the recursive generation of a list: Starting from some base case,
                                                                                                                                | this command generates an infinite list with the pattern function given.
                                                                                                                                £ | Flag for λ. Instead of outputting an infinite stream, only print the first n.
                                                                                                                                ₂₃+ | Add a(n-2) and a(n-3).





                                                                                                                                share|improve this answer











                                                                                                                                $endgroup$













                                                                                                                                • $begingroup$
                                                                                                                                  I don't think 1Ð) can be 2 bytes tbh. I can think of six different 3-bytes alternatives, but no 2-byters.
                                                                                                                                  $endgroup$
                                                                                                                                  – Kevin Cruijssen
                                                                                                                                  19 hours ago














                                                                                                                                2












                                                                                                                                2








                                                                                                                                2





                                                                                                                                $begingroup$


                                                                                                                                05AB1E, 8 bytes



                                                                                                                                1Ð)λ£₂₃+


                                                                                                                                Try it online!



                                                                                                                                Bear with me, I haven't golfed in a while. I wonder if there's a shorter substitute for 1Ð) which works in this case (I've tried 1D), 3Å1 etc. but none of them save bytes). Outputs the first $n$ terms of the sequence. Or, without the £, it would output an infinite stream of the terms of the sequence.



                                                                                                                                How?



                                                                                                                                1Ð)λ£₂₃+ | Full program.
                                                                                                                                1Ð) | Initialize the stack with [1, 1, 1].
                                                                                                                                λ | Begin the recursive generation of a list: Starting from some base case,
                                                                                                                                | this command generates an infinite list with the pattern function given.
                                                                                                                                £ | Flag for λ. Instead of outputting an infinite stream, only print the first n.
                                                                                                                                ₂₃+ | Add a(n-2) and a(n-3).





                                                                                                                                share|improve this answer











                                                                                                                                $endgroup$




                                                                                                                                05AB1E, 8 bytes



                                                                                                                                1Ð)λ£₂₃+


                                                                                                                                Try it online!



                                                                                                                                Bear with me, I haven't golfed in a while. I wonder if there's a shorter substitute for 1Ð) which works in this case (I've tried 1D), 3Å1 etc. but none of them save bytes). Outputs the first $n$ terms of the sequence. Or, without the £, it would output an infinite stream of the terms of the sequence.



                                                                                                                                How?



                                                                                                                                1Ð)λ£₂₃+ | Full program.
                                                                                                                                1Ð) | Initialize the stack with [1, 1, 1].
                                                                                                                                λ | Begin the recursive generation of a list: Starting from some base case,
                                                                                                                                | this command generates an infinite list with the pattern function given.
                                                                                                                                £ | Flag for λ. Instead of outputting an infinite stream, only print the first n.
                                                                                                                                ₂₃+ | Add a(n-2) and a(n-3).






                                                                                                                                share|improve this answer














                                                                                                                                share|improve this answer



                                                                                                                                share|improve this answer








                                                                                                                                edited 23 hours ago

























                                                                                                                                answered 23 hours ago









                                                                                                                                Mr. XcoderMr. Xcoder

                                                                                                                                32.2k759200




                                                                                                                                32.2k759200












                                                                                                                                • $begingroup$
                                                                                                                                  I don't think 1Ð) can be 2 bytes tbh. I can think of six different 3-bytes alternatives, but no 2-byters.
                                                                                                                                  $endgroup$
                                                                                                                                  – Kevin Cruijssen
                                                                                                                                  19 hours ago


















                                                                                                                                • $begingroup$
                                                                                                                                  I don't think 1Ð) can be 2 bytes tbh. I can think of six different 3-bytes alternatives, but no 2-byters.
                                                                                                                                  $endgroup$
                                                                                                                                  – Kevin Cruijssen
                                                                                                                                  19 hours ago
















                                                                                                                                $begingroup$
                                                                                                                                I don't think 1Ð) can be 2 bytes tbh. I can think of six different 3-bytes alternatives, but no 2-byters.
                                                                                                                                $endgroup$
                                                                                                                                – Kevin Cruijssen
                                                                                                                                19 hours ago




                                                                                                                                $begingroup$
                                                                                                                                I don't think 1Ð) can be 2 bytes tbh. I can think of six different 3-bytes alternatives, but no 2-byters.
                                                                                                                                $endgroup$
                                                                                                                                – Kevin Cruijssen
                                                                                                                                19 hours ago











                                                                                                                                2












                                                                                                                                $begingroup$


                                                                                                                                R + pryr, 38 36 bytes



                                                                                                                                Zero-indexed recursive function.





                                                                                                                                f=pryr::f(`if`(n<3,1,f(n-2)+f(n-3)))


                                                                                                                                Try it online!



                                                                                                                                Thanks to @Giuseppe for pointing out two obviously needless bytes.






                                                                                                                                share|improve this answer











                                                                                                                                $endgroup$









                                                                                                                                • 1




                                                                                                                                  $begingroup$
                                                                                                                                  If you're going to be using pryr, the language should be R + pryr and this can be 36 bytes
                                                                                                                                  $endgroup$
                                                                                                                                  – Giuseppe
                                                                                                                                  9 hours ago












                                                                                                                                • $begingroup$
                                                                                                                                  @Giuseppe thanks! Updated now.
                                                                                                                                  $endgroup$
                                                                                                                                  – rturnbull
                                                                                                                                  8 hours ago
















                                                                                                                                2












                                                                                                                                $begingroup$


                                                                                                                                R + pryr, 38 36 bytes



                                                                                                                                Zero-indexed recursive function.





                                                                                                                                f=pryr::f(`if`(n<3,1,f(n-2)+f(n-3)))


                                                                                                                                Try it online!



                                                                                                                                Thanks to @Giuseppe for pointing out two obviously needless bytes.






                                                                                                                                share|improve this answer











                                                                                                                                $endgroup$









                                                                                                                                • 1




                                                                                                                                  $begingroup$
                                                                                                                                  If you're going to be using pryr, the language should be R + pryr and this can be 36 bytes
                                                                                                                                  $endgroup$
                                                                                                                                  – Giuseppe
                                                                                                                                  9 hours ago












                                                                                                                                • $begingroup$
                                                                                                                                  @Giuseppe thanks! Updated now.
                                                                                                                                  $endgroup$
                                                                                                                                  – rturnbull
                                                                                                                                  8 hours ago














                                                                                                                                2












                                                                                                                                2








                                                                                                                                2





                                                                                                                                $begingroup$


                                                                                                                                R + pryr, 38 36 bytes



                                                                                                                                Zero-indexed recursive function.





                                                                                                                                f=pryr::f(`if`(n<3,1,f(n-2)+f(n-3)))


                                                                                                                                Try it online!



                                                                                                                                Thanks to @Giuseppe for pointing out two obviously needless bytes.






                                                                                                                                share|improve this answer











                                                                                                                                $endgroup$




                                                                                                                                R + pryr, 38 36 bytes



                                                                                                                                Zero-indexed recursive function.





                                                                                                                                f=pryr::f(`if`(n<3,1,f(n-2)+f(n-3)))


                                                                                                                                Try it online!



                                                                                                                                Thanks to @Giuseppe for pointing out two obviously needless bytes.







                                                                                                                                share|improve this answer














                                                                                                                                share|improve this answer



                                                                                                                                share|improve this answer








                                                                                                                                edited 8 hours ago

























                                                                                                                                answered 10 hours ago









                                                                                                                                rturnbullrturnbull

                                                                                                                                3,499925




                                                                                                                                3,499925








                                                                                                                                • 1




                                                                                                                                  $begingroup$
                                                                                                                                  If you're going to be using pryr, the language should be R + pryr and this can be 36 bytes
                                                                                                                                  $endgroup$
                                                                                                                                  – Giuseppe
                                                                                                                                  9 hours ago












                                                                                                                                • $begingroup$
                                                                                                                                  @Giuseppe thanks! Updated now.
                                                                                                                                  $endgroup$
                                                                                                                                  – rturnbull
                                                                                                                                  8 hours ago














                                                                                                                                • 1




                                                                                                                                  $begingroup$
                                                                                                                                  If you're going to be using pryr, the language should be R + pryr and this can be 36 bytes
                                                                                                                                  $endgroup$
                                                                                                                                  – Giuseppe
                                                                                                                                  9 hours ago












                                                                                                                                • $begingroup$
                                                                                                                                  @Giuseppe thanks! Updated now.
                                                                                                                                  $endgroup$
                                                                                                                                  – rturnbull
                                                                                                                                  8 hours ago








                                                                                                                                1




                                                                                                                                1




                                                                                                                                $begingroup$
                                                                                                                                If you're going to be using pryr, the language should be R + pryr and this can be 36 bytes
                                                                                                                                $endgroup$
                                                                                                                                – Giuseppe
                                                                                                                                9 hours ago






                                                                                                                                $begingroup$
                                                                                                                                If you're going to be using pryr, the language should be R + pryr and this can be 36 bytes
                                                                                                                                $endgroup$
                                                                                                                                – Giuseppe
                                                                                                                                9 hours ago














                                                                                                                                $begingroup$
                                                                                                                                @Giuseppe thanks! Updated now.
                                                                                                                                $endgroup$
                                                                                                                                – rturnbull
                                                                                                                                8 hours ago




                                                                                                                                $begingroup$
                                                                                                                                @Giuseppe thanks! Updated now.
                                                                                                                                $endgroup$
                                                                                                                                – rturnbull
                                                                                                                                8 hours ago











                                                                                                                                1












                                                                                                                                $begingroup$


                                                                                                                                C# (Visual C# Interactive Compiler), 34 bytes





                                                                                                                                int f(int g)=>g<3?1:f(g-2)+f(g-3);


                                                                                                                                Try it online!






                                                                                                                                share|improve this answer









                                                                                                                                $endgroup$


















                                                                                                                                  1












                                                                                                                                  $begingroup$


                                                                                                                                  C# (Visual C# Interactive Compiler), 34 bytes





                                                                                                                                  int f(int g)=>g<3?1:f(g-2)+f(g-3);


                                                                                                                                  Try it online!






                                                                                                                                  share|improve this answer









                                                                                                                                  $endgroup$
















                                                                                                                                    1












                                                                                                                                    1








                                                                                                                                    1





                                                                                                                                    $begingroup$


                                                                                                                                    C# (Visual C# Interactive Compiler), 34 bytes





                                                                                                                                    int f(int g)=>g<3?1:f(g-2)+f(g-3);


                                                                                                                                    Try it online!






                                                                                                                                    share|improve this answer









                                                                                                                                    $endgroup$




                                                                                                                                    C# (Visual C# Interactive Compiler), 34 bytes





                                                                                                                                    int f(int g)=>g<3?1:f(g-2)+f(g-3);


                                                                                                                                    Try it online!







                                                                                                                                    share|improve this answer












                                                                                                                                    share|improve this answer



                                                                                                                                    share|improve this answer










                                                                                                                                    answered yesterday









                                                                                                                                    Embodiment of IgnoranceEmbodiment of Ignorance

                                                                                                                                    2,866127




                                                                                                                                    2,866127























                                                                                                                                        1












                                                                                                                                        $begingroup$

                                                                                                                                        TI-BASIC (TI-84), 34 bytes



                                                                                                                                        [[0,1,0][0,0,1][1,1,0]]^(Ans+5:Ans(1,1


                                                                                                                                        0-indexed $N$th term of the sequence.



                                                                                                                                        Input is in Ans.

                                                                                                                                        Output is in Ans and is automatically printed out.



                                                                                                                                        I figured that enough time had passed, plus multiple answers had been posted, of which there were many which out-golfed this answer.



                                                                                                                                        Example:



                                                                                                                                        0
                                                                                                                                        0
                                                                                                                                        prgmCDGFD
                                                                                                                                        1
                                                                                                                                        9
                                                                                                                                        9
                                                                                                                                        prgmCDGFD
                                                                                                                                        9
                                                                                                                                        16
                                                                                                                                        16
                                                                                                                                        prgmCDGFD
                                                                                                                                        65


                                                                                                                                        Explanation:



                                                                                                                                        [[0,1,0][0,0,1][1,1,0]]^(Ans+5:Ans(1,1      ;full program (example input: 6)

                                                                                                                                        [[0,1,0][0,0,1][1,1,0]] ;generate the following matrix:
                                                                                                                                        ; [0 1 0]
                                                                                                                                        ; [0 0 1]
                                                                                                                                        ; [1 1 0]
                                                                                                                                        ^(Ans+5 ;then raise it to the power of: input + 5
                                                                                                                                        ; [4 7 5]
                                                                                                                                        ; [5 9 7]
                                                                                                                                        ; [7 12 9]
                                                                                                                                        Ans(1,1 ;get the top-left index and leave it in "Ans"
                                                                                                                                        ;implicitly print Ans





                                                                                                                                        share|improve this answer









                                                                                                                                        $endgroup$


















                                                                                                                                          1












                                                                                                                                          $begingroup$

                                                                                                                                          TI-BASIC (TI-84), 34 bytes



                                                                                                                                          [[0,1,0][0,0,1][1,1,0]]^(Ans+5:Ans(1,1


                                                                                                                                          0-indexed $N$th term of the sequence.



                                                                                                                                          Input is in Ans.

                                                                                                                                          Output is in Ans and is automatically printed out.



                                                                                                                                          I figured that enough time had passed, plus multiple answers had been posted, of which there were many which out-golfed this answer.



                                                                                                                                          Example:



                                                                                                                                          0
                                                                                                                                          0
                                                                                                                                          prgmCDGFD
                                                                                                                                          1
                                                                                                                                          9
                                                                                                                                          9
                                                                                                                                          prgmCDGFD
                                                                                                                                          9
                                                                                                                                          16
                                                                                                                                          16
                                                                                                                                          prgmCDGFD
                                                                                                                                          65


                                                                                                                                          Explanation:



                                                                                                                                          [[0,1,0][0,0,1][1,1,0]]^(Ans+5:Ans(1,1      ;full program (example input: 6)

                                                                                                                                          [[0,1,0][0,0,1][1,1,0]] ;generate the following matrix:
                                                                                                                                          ; [0 1 0]
                                                                                                                                          ; [0 0 1]
                                                                                                                                          ; [1 1 0]
                                                                                                                                          ^(Ans+5 ;then raise it to the power of: input + 5
                                                                                                                                          ; [4 7 5]
                                                                                                                                          ; [5 9 7]
                                                                                                                                          ; [7 12 9]
                                                                                                                                          Ans(1,1 ;get the top-left index and leave it in "Ans"
                                                                                                                                          ;implicitly print Ans





                                                                                                                                          share|improve this answer









                                                                                                                                          $endgroup$
















                                                                                                                                            1












                                                                                                                                            1








                                                                                                                                            1





                                                                                                                                            $begingroup$

                                                                                                                                            TI-BASIC (TI-84), 34 bytes



                                                                                                                                            [[0,1,0][0,0,1][1,1,0]]^(Ans+5:Ans(1,1


                                                                                                                                            0-indexed $N$th term of the sequence.



                                                                                                                                            Input is in Ans.

                                                                                                                                            Output is in Ans and is automatically printed out.



                                                                                                                                            I figured that enough time had passed, plus multiple answers had been posted, of which there were many which out-golfed this answer.



                                                                                                                                            Example:



                                                                                                                                            0
                                                                                                                                            0
                                                                                                                                            prgmCDGFD
                                                                                                                                            1
                                                                                                                                            9
                                                                                                                                            9
                                                                                                                                            prgmCDGFD
                                                                                                                                            9
                                                                                                                                            16
                                                                                                                                            16
                                                                                                                                            prgmCDGFD
                                                                                                                                            65


                                                                                                                                            Explanation:



                                                                                                                                            [[0,1,0][0,0,1][1,1,0]]^(Ans+5:Ans(1,1      ;full program (example input: 6)

                                                                                                                                            [[0,1,0][0,0,1][1,1,0]] ;generate the following matrix:
                                                                                                                                            ; [0 1 0]
                                                                                                                                            ; [0 0 1]
                                                                                                                                            ; [1 1 0]
                                                                                                                                            ^(Ans+5 ;then raise it to the power of: input + 5
                                                                                                                                            ; [4 7 5]
                                                                                                                                            ; [5 9 7]
                                                                                                                                            ; [7 12 9]
                                                                                                                                            Ans(1,1 ;get the top-left index and leave it in "Ans"
                                                                                                                                            ;implicitly print Ans





                                                                                                                                            share|improve this answer









                                                                                                                                            $endgroup$



                                                                                                                                            TI-BASIC (TI-84), 34 bytes



                                                                                                                                            [[0,1,0][0,0,1][1,1,0]]^(Ans+5:Ans(1,1


                                                                                                                                            0-indexed $N$th term of the sequence.



                                                                                                                                            Input is in Ans.

                                                                                                                                            Output is in Ans and is automatically printed out.



                                                                                                                                            I figured that enough time had passed, plus multiple answers had been posted, of which there were many which out-golfed this answer.



                                                                                                                                            Example:



                                                                                                                                            0
                                                                                                                                            0
                                                                                                                                            prgmCDGFD
                                                                                                                                            1
                                                                                                                                            9
                                                                                                                                            9
                                                                                                                                            prgmCDGFD
                                                                                                                                            9
                                                                                                                                            16
                                                                                                                                            16
                                                                                                                                            prgmCDGFD
                                                                                                                                            65


                                                                                                                                            Explanation:



                                                                                                                                            [[0,1,0][0,0,1][1,1,0]]^(Ans+5:Ans(1,1      ;full program (example input: 6)

                                                                                                                                            [[0,1,0][0,0,1][1,1,0]] ;generate the following matrix:
                                                                                                                                            ; [0 1 0]
                                                                                                                                            ; [0 0 1]
                                                                                                                                            ; [1 1 0]
                                                                                                                                            ^(Ans+5 ;then raise it to the power of: input + 5
                                                                                                                                            ; [4 7 5]
                                                                                                                                            ; [5 9 7]
                                                                                                                                            ; [7 12 9]
                                                                                                                                            Ans(1,1 ;get the top-left index and leave it in "Ans"
                                                                                                                                            ;implicitly print Ans






                                                                                                                                            share|improve this answer












                                                                                                                                            share|improve this answer



                                                                                                                                            share|improve this answer










                                                                                                                                            answered yesterday









                                                                                                                                            TauTau

                                                                                                                                            911515




                                                                                                                                            911515























                                                                                                                                                1












                                                                                                                                                $begingroup$


                                                                                                                                                Ruby, 26 bytes





                                                                                                                                                f=->n{n<3?1:f[n-2]+f[n-3]}


                                                                                                                                                Try it online!






                                                                                                                                                share|improve this answer









                                                                                                                                                $endgroup$


















                                                                                                                                                  1












                                                                                                                                                  $begingroup$


                                                                                                                                                  Ruby, 26 bytes





                                                                                                                                                  f=->n{n<3?1:f[n-2]+f[n-3]}


                                                                                                                                                  Try it online!






                                                                                                                                                  share|improve this answer









                                                                                                                                                  $endgroup$
















                                                                                                                                                    1












                                                                                                                                                    1








                                                                                                                                                    1





                                                                                                                                                    $begingroup$


                                                                                                                                                    Ruby, 26 bytes





                                                                                                                                                    f=->n{n<3?1:f[n-2]+f[n-3]}


                                                                                                                                                    Try it online!






                                                                                                                                                    share|improve this answer









                                                                                                                                                    $endgroup$




                                                                                                                                                    Ruby, 26 bytes





                                                                                                                                                    f=->n{n<3?1:f[n-2]+f[n-3]}


                                                                                                                                                    Try it online!







                                                                                                                                                    share|improve this answer












                                                                                                                                                    share|improve this answer



                                                                                                                                                    share|improve this answer










                                                                                                                                                    answered 22 hours ago









                                                                                                                                                    G BG B

                                                                                                                                                    8,2161429




                                                                                                                                                    8,2161429























                                                                                                                                                        1












                                                                                                                                                        $begingroup$


                                                                                                                                                        Japt, 12 bytes



                                                                                                                                                        Returns the first n terms, 0-indexed. Replace h with g to return the nth term, 1-indexed.



                                                                                                                                                        ÈnZs3n)x}hBì


                                                                                                                                                        Try it



                                                                                                                                                        ÈnZs3n)x}hBì     :Implicit input of integer U
                                                                                                                                                        B :11
                                                                                                                                                        ì :Convert to digit array
                                                                                                                                                        h :Repeat the following until the length of the array is U, pushing the result to the array each time
                                                                                                                                                        È : Take the last element X from the array Z and pass it through the following function
                                                                                                                                                        n : Subtract X from
                                                                                                                                                        Zs : Slice Z
                                                                                                                                                        3n : -3, giving the last 3 elements in the array
                                                                                                                                                        ) : End slice
                                                                                                                                                        x : Reduce by addition
                                                                                                                                                        } : End function





                                                                                                                                                        share|improve this answer











                                                                                                                                                        $endgroup$


















                                                                                                                                                          1












                                                                                                                                                          $begingroup$


                                                                                                                                                          Japt, 12 bytes



                                                                                                                                                          Returns the first n terms, 0-indexed. Replace h with g to return the nth term, 1-indexed.



                                                                                                                                                          ÈnZs3n)x}hBì


                                                                                                                                                          Try it



                                                                                                                                                          ÈnZs3n)x}hBì     :Implicit input of integer U
                                                                                                                                                          B :11
                                                                                                                                                          ì :Convert to digit array
                                                                                                                                                          h :Repeat the following until the length of the array is U, pushing the result to the array each time
                                                                                                                                                          È : Take the last element X from the array Z and pass it through the following function
                                                                                                                                                          n : Subtract X from
                                                                                                                                                          Zs : Slice Z
                                                                                                                                                          3n : -3, giving the last 3 elements in the array
                                                                                                                                                          ) : End slice
                                                                                                                                                          x : Reduce by addition
                                                                                                                                                          } : End function





                                                                                                                                                          share|improve this answer











                                                                                                                                                          $endgroup$
















                                                                                                                                                            1












                                                                                                                                                            1








                                                                                                                                                            1





                                                                                                                                                            $begingroup$


                                                                                                                                                            Japt, 12 bytes



                                                                                                                                                            Returns the first n terms, 0-indexed. Replace h with g to return the nth term, 1-indexed.



                                                                                                                                                            ÈnZs3n)x}hBì


                                                                                                                                                            Try it



                                                                                                                                                            ÈnZs3n)x}hBì     :Implicit input of integer U
                                                                                                                                                            B :11
                                                                                                                                                            ì :Convert to digit array
                                                                                                                                                            h :Repeat the following until the length of the array is U, pushing the result to the array each time
                                                                                                                                                            È : Take the last element X from the array Z and pass it through the following function
                                                                                                                                                            n : Subtract X from
                                                                                                                                                            Zs : Slice Z
                                                                                                                                                            3n : -3, giving the last 3 elements in the array
                                                                                                                                                            ) : End slice
                                                                                                                                                            x : Reduce by addition
                                                                                                                                                            } : End function





                                                                                                                                                            share|improve this answer











                                                                                                                                                            $endgroup$




                                                                                                                                                            Japt, 12 bytes



                                                                                                                                                            Returns the first n terms, 0-indexed. Replace h with g to return the nth term, 1-indexed.



                                                                                                                                                            ÈnZs3n)x}hBì


                                                                                                                                                            Try it



                                                                                                                                                            ÈnZs3n)x}hBì     :Implicit input of integer U
                                                                                                                                                            B :11
                                                                                                                                                            ì :Convert to digit array
                                                                                                                                                            h :Repeat the following until the length of the array is U, pushing the result to the array each time
                                                                                                                                                            È : Take the last element X from the array Z and pass it through the following function
                                                                                                                                                            n : Subtract X from
                                                                                                                                                            Zs : Slice Z
                                                                                                                                                            3n : -3, giving the last 3 elements in the array
                                                                                                                                                            ) : End slice
                                                                                                                                                            x : Reduce by addition
                                                                                                                                                            } : End function






                                                                                                                                                            share|improve this answer














                                                                                                                                                            share|improve this answer



                                                                                                                                                            share|improve this answer








                                                                                                                                                            edited 20 hours ago

























                                                                                                                                                            answered yesterday









                                                                                                                                                            ShaggyShaggy

                                                                                                                                                            18.9k21768




                                                                                                                                                            18.9k21768























                                                                                                                                                                1












                                                                                                                                                                $begingroup$


                                                                                                                                                                Perl 5, 34 bytes





                                                                                                                                                                sub f{"@_"<3||f("@_"-2)+f("@_"-3)}


                                                                                                                                                                Try it online!






                                                                                                                                                                share|improve this answer









                                                                                                                                                                $endgroup$


















                                                                                                                                                                  1












                                                                                                                                                                  $begingroup$


                                                                                                                                                                  Perl 5, 34 bytes





                                                                                                                                                                  sub f{"@_"<3||f("@_"-2)+f("@_"-3)}


                                                                                                                                                                  Try it online!






                                                                                                                                                                  share|improve this answer









                                                                                                                                                                  $endgroup$
















                                                                                                                                                                    1












                                                                                                                                                                    1








                                                                                                                                                                    1





                                                                                                                                                                    $begingroup$


                                                                                                                                                                    Perl 5, 34 bytes





                                                                                                                                                                    sub f{"@_"<3||f("@_"-2)+f("@_"-3)}


                                                                                                                                                                    Try it online!






                                                                                                                                                                    share|improve this answer









                                                                                                                                                                    $endgroup$




                                                                                                                                                                    Perl 5, 34 bytes





                                                                                                                                                                    sub f{"@_"<3||f("@_"-2)+f("@_"-3)}


                                                                                                                                                                    Try it online!







                                                                                                                                                                    share|improve this answer












                                                                                                                                                                    share|improve this answer



                                                                                                                                                                    share|improve this answer










                                                                                                                                                                    answered 12 hours ago









                                                                                                                                                                    XcaliXcali

                                                                                                                                                                    5,465520




                                                                                                                                                                    5,465520























                                                                                                                                                                        1












                                                                                                                                                                        $begingroup$


                                                                                                                                                                        C++ (gcc), 81 75 bytes



                                                                                                                                                                        -6 bytes to small golfing





                                                                                                                                                                        int a(int n){int a=1,b=1,c=1,d,i=2;for(;i++<n;)d=a+b,a=b,b=c,c=d;return c;}


                                                                                                                                                                        Try it online!



                                                                                                                                                                        Simple function to compute the values iteratively. No loop occurs for n<3, so the first cases default to the initial 1.






                                                                                                                                                                        share|improve this answer











                                                                                                                                                                        $endgroup$


















                                                                                                                                                                          1












                                                                                                                                                                          $begingroup$


                                                                                                                                                                          C++ (gcc), 81 75 bytes



                                                                                                                                                                          -6 bytes to small golfing





                                                                                                                                                                          int a(int n){int a=1,b=1,c=1,d,i=2;for(;i++<n;)d=a+b,a=b,b=c,c=d;return c;}


                                                                                                                                                                          Try it online!



                                                                                                                                                                          Simple function to compute the values iteratively. No loop occurs for n<3, so the first cases default to the initial 1.






                                                                                                                                                                          share|improve this answer











                                                                                                                                                                          $endgroup$
















                                                                                                                                                                            1












                                                                                                                                                                            1








                                                                                                                                                                            1





                                                                                                                                                                            $begingroup$


                                                                                                                                                                            C++ (gcc), 81 75 bytes



                                                                                                                                                                            -6 bytes to small golfing





                                                                                                                                                                            int a(int n){int a=1,b=1,c=1,d,i=2;for(;i++<n;)d=a+b,a=b,b=c,c=d;return c;}


                                                                                                                                                                            Try it online!



                                                                                                                                                                            Simple function to compute the values iteratively. No loop occurs for n<3, so the first cases default to the initial 1.






                                                                                                                                                                            share|improve this answer











                                                                                                                                                                            $endgroup$




                                                                                                                                                                            C++ (gcc), 81 75 bytes



                                                                                                                                                                            -6 bytes to small golfing





                                                                                                                                                                            int a(int n){int a=1,b=1,c=1,d,i=2;for(;i++<n;)d=a+b,a=b,b=c,c=d;return c;}


                                                                                                                                                                            Try it online!



                                                                                                                                                                            Simple function to compute the values iteratively. No loop occurs for n<3, so the first cases default to the initial 1.







                                                                                                                                                                            share|improve this answer














                                                                                                                                                                            share|improve this answer



                                                                                                                                                                            share|improve this answer








                                                                                                                                                                            edited 10 hours ago

























                                                                                                                                                                            answered 10 hours ago









                                                                                                                                                                            Neil A.Neil A.

                                                                                                                                                                            1,348120




                                                                                                                                                                            1,348120























                                                                                                                                                                                1












                                                                                                                                                                                $begingroup$

                                                                                                                                                                                Java, 41 bytes



                                                                                                                                                                                Can't use a lambda (runtime error). Port of this Javascript answer



                                                                                                                                                                                int f(int n){return n<3?1:f(n-2)+f(n-3);}


                                                                                                                                                                                TIO






                                                                                                                                                                                share|improve this answer









                                                                                                                                                                                $endgroup$


















                                                                                                                                                                                  1












                                                                                                                                                                                  $begingroup$

                                                                                                                                                                                  Java, 41 bytes



                                                                                                                                                                                  Can't use a lambda (runtime error). Port of this Javascript answer



                                                                                                                                                                                  int f(int n){return n<3?1:f(n-2)+f(n-3);}


                                                                                                                                                                                  TIO






                                                                                                                                                                                  share|improve this answer









                                                                                                                                                                                  $endgroup$
















                                                                                                                                                                                    1












                                                                                                                                                                                    1








                                                                                                                                                                                    1





                                                                                                                                                                                    $begingroup$

                                                                                                                                                                                    Java, 41 bytes



                                                                                                                                                                                    Can't use a lambda (runtime error). Port of this Javascript answer



                                                                                                                                                                                    int f(int n){return n<3?1:f(n-2)+f(n-3);}


                                                                                                                                                                                    TIO






                                                                                                                                                                                    share|improve this answer









                                                                                                                                                                                    $endgroup$



                                                                                                                                                                                    Java, 41 bytes



                                                                                                                                                                                    Can't use a lambda (runtime error). Port of this Javascript answer



                                                                                                                                                                                    int f(int n){return n<3?1:f(n-2)+f(n-3);}


                                                                                                                                                                                    TIO







                                                                                                                                                                                    share|improve this answer












                                                                                                                                                                                    share|improve this answer



                                                                                                                                                                                    share|improve this answer










                                                                                                                                                                                    answered 9 hours ago









                                                                                                                                                                                    Benjamin UrquhartBenjamin Urquhart

                                                                                                                                                                                    37017




                                                                                                                                                                                    37017























                                                                                                                                                                                        1












                                                                                                                                                                                        $begingroup$

                                                                                                                                                                                        x86 32-bit machine code, 17 bytes



                                                                                                                                                                                        53 33 db f7 e3 43 83 c1 04 03 d8 93 92 e2 fa 5b c3


                                                                                                                                                                                        Disassembly:



                                                                                                                                                                                        00CE1250 53                   push        ebx  
                                                                                                                                                                                        00CE1251 33 DB xor ebx,ebx
                                                                                                                                                                                        00CE1253 F7 E3 mul eax,ebx
                                                                                                                                                                                        00CE1255 43 inc ebx
                                                                                                                                                                                        00CE1256 83 C1 04 add ecx,4
                                                                                                                                                                                        00CE1259 03 D8 add ebx,eax
                                                                                                                                                                                        00CE125B 93 xchg eax,ebx
                                                                                                                                                                                        00CE125C 92 xchg eax,edx
                                                                                                                                                                                        00CE125D E2 FA loop myloop (0CE1259h)
                                                                                                                                                                                        00CE125F 5B pop ebx
                                                                                                                                                                                        00CE1260 C3 ret


                                                                                                                                                                                        It is 0-indexed. The initialization is conveniently achieved by calculating eax * 0. The 128-bit result is 0, and it goes in edx:eax.



                                                                                                                                                                                        At the beginning of each iteration, the order of the registers is ebx, eax, edx. I had to choose the right order to take advantage of the encoding for the xchg eax instruction - 1 byte.



                                                                                                                                                                                        I had to add 4 to the loop counter in order to let the output reach eax, which holds the function's return value in the fastcall convention.



                                                                                                                                                                                        I could use some other calling convention, which doesn't require saving and restoring ebx, but fastcall is fun anyway :)






                                                                                                                                                                                        share|improve this answer









                                                                                                                                                                                        $endgroup$













                                                                                                                                                                                        • $begingroup$
                                                                                                                                                                                          I love to see machine code answers on PP&CG! +1
                                                                                                                                                                                          $endgroup$
                                                                                                                                                                                          – Tau
                                                                                                                                                                                          3 hours ago
















                                                                                                                                                                                        1












                                                                                                                                                                                        $begingroup$

                                                                                                                                                                                        x86 32-bit machine code, 17 bytes



                                                                                                                                                                                        53 33 db f7 e3 43 83 c1 04 03 d8 93 92 e2 fa 5b c3


                                                                                                                                                                                        Disassembly:



                                                                                                                                                                                        00CE1250 53                   push        ebx  
                                                                                                                                                                                        00CE1251 33 DB xor ebx,ebx
                                                                                                                                                                                        00CE1253 F7 E3 mul eax,ebx
                                                                                                                                                                                        00CE1255 43 inc ebx
                                                                                                                                                                                        00CE1256 83 C1 04 add ecx,4
                                                                                                                                                                                        00CE1259 03 D8 add ebx,eax
                                                                                                                                                                                        00CE125B 93 xchg eax,ebx
                                                                                                                                                                                        00CE125C 92 xchg eax,edx
                                                                                                                                                                                        00CE125D E2 FA loop myloop (0CE1259h)
                                                                                                                                                                                        00CE125F 5B pop ebx
                                                                                                                                                                                        00CE1260 C3 ret


                                                                                                                                                                                        It is 0-indexed. The initialization is conveniently achieved by calculating eax * 0. The 128-bit result is 0, and it goes in edx:eax.



                                                                                                                                                                                        At the beginning of each iteration, the order of the registers is ebx, eax, edx. I had to choose the right order to take advantage of the encoding for the xchg eax instruction - 1 byte.



                                                                                                                                                                                        I had to add 4 to the loop counter in order to let the output reach eax, which holds the function's return value in the fastcall convention.



                                                                                                                                                                                        I could use some other calling convention, which doesn't require saving and restoring ebx, but fastcall is fun anyway :)






                                                                                                                                                                                        share|improve this answer









                                                                                                                                                                                        $endgroup$













                                                                                                                                                                                        • $begingroup$
                                                                                                                                                                                          I love to see machine code answers on PP&CG! +1
                                                                                                                                                                                          $endgroup$
                                                                                                                                                                                          – Tau
                                                                                                                                                                                          3 hours ago














                                                                                                                                                                                        1












                                                                                                                                                                                        1








                                                                                                                                                                                        1





                                                                                                                                                                                        $begingroup$

                                                                                                                                                                                        x86 32-bit machine code, 17 bytes



                                                                                                                                                                                        53 33 db f7 e3 43 83 c1 04 03 d8 93 92 e2 fa 5b c3


                                                                                                                                                                                        Disassembly:



                                                                                                                                                                                        00CE1250 53                   push        ebx  
                                                                                                                                                                                        00CE1251 33 DB xor ebx,ebx
                                                                                                                                                                                        00CE1253 F7 E3 mul eax,ebx
                                                                                                                                                                                        00CE1255 43 inc ebx
                                                                                                                                                                                        00CE1256 83 C1 04 add ecx,4
                                                                                                                                                                                        00CE1259 03 D8 add ebx,eax
                                                                                                                                                                                        00CE125B 93 xchg eax,ebx
                                                                                                                                                                                        00CE125C 92 xchg eax,edx
                                                                                                                                                                                        00CE125D E2 FA loop myloop (0CE1259h)
                                                                                                                                                                                        00CE125F 5B pop ebx
                                                                                                                                                                                        00CE1260 C3 ret


                                                                                                                                                                                        It is 0-indexed. The initialization is conveniently achieved by calculating eax * 0. The 128-bit result is 0, and it goes in edx:eax.



                                                                                                                                                                                        At the beginning of each iteration, the order of the registers is ebx, eax, edx. I had to choose the right order to take advantage of the encoding for the xchg eax instruction - 1 byte.



                                                                                                                                                                                        I had to add 4 to the loop counter in order to let the output reach eax, which holds the function's return value in the fastcall convention.



                                                                                                                                                                                        I could use some other calling convention, which doesn't require saving and restoring ebx, but fastcall is fun anyway :)






                                                                                                                                                                                        share|improve this answer









                                                                                                                                                                                        $endgroup$



                                                                                                                                                                                        x86 32-bit machine code, 17 bytes



                                                                                                                                                                                        53 33 db f7 e3 43 83 c1 04 03 d8 93 92 e2 fa 5b c3


                                                                                                                                                                                        Disassembly:



                                                                                                                                                                                        00CE1250 53                   push        ebx  
                                                                                                                                                                                        00CE1251 33 DB xor ebx,ebx
                                                                                                                                                                                        00CE1253 F7 E3 mul eax,ebx
                                                                                                                                                                                        00CE1255 43 inc ebx
                                                                                                                                                                                        00CE1256 83 C1 04 add ecx,4
                                                                                                                                                                                        00CE1259 03 D8 add ebx,eax
                                                                                                                                                                                        00CE125B 93 xchg eax,ebx
                                                                                                                                                                                        00CE125C 92 xchg eax,edx
                                                                                                                                                                                        00CE125D E2 FA loop myloop (0CE1259h)
                                                                                                                                                                                        00CE125F 5B pop ebx
                                                                                                                                                                                        00CE1260 C3 ret


                                                                                                                                                                                        It is 0-indexed. The initialization is conveniently achieved by calculating eax * 0. The 128-bit result is 0, and it goes in edx:eax.



                                                                                                                                                                                        At the beginning of each iteration, the order of the registers is ebx, eax, edx. I had to choose the right order to take advantage of the encoding for the xchg eax instruction - 1 byte.



                                                                                                                                                                                        I had to add 4 to the loop counter in order to let the output reach eax, which holds the function's return value in the fastcall convention.



                                                                                                                                                                                        I could use some other calling convention, which doesn't require saving and restoring ebx, but fastcall is fun anyway :)







                                                                                                                                                                                        share|improve this answer












                                                                                                                                                                                        share|improve this answer



                                                                                                                                                                                        share|improve this answer










                                                                                                                                                                                        answered 8 hours ago









                                                                                                                                                                                        anatolyganatolyg

                                                                                                                                                                                        7,2392166




                                                                                                                                                                                        7,2392166












                                                                                                                                                                                        • $begingroup$
                                                                                                                                                                                          I love to see machine code answers on PP&CG! +1
                                                                                                                                                                                          $endgroup$
                                                                                                                                                                                          – Tau
                                                                                                                                                                                          3 hours ago


















                                                                                                                                                                                        • $begingroup$
                                                                                                                                                                                          I love to see machine code answers on PP&CG! +1
                                                                                                                                                                                          $endgroup$
                                                                                                                                                                                          – Tau
                                                                                                                                                                                          3 hours ago
















                                                                                                                                                                                        $begingroup$
                                                                                                                                                                                        I love to see machine code answers on PP&CG! +1
                                                                                                                                                                                        $endgroup$
                                                                                                                                                                                        – Tau
                                                                                                                                                                                        3 hours ago




                                                                                                                                                                                        $begingroup$
                                                                                                                                                                                        I love to see machine code answers on PP&CG! +1
                                                                                                                                                                                        $endgroup$
                                                                                                                                                                                        – Tau
                                                                                                                                                                                        3 hours ago











                                                                                                                                                                                        1












                                                                                                                                                                                        $begingroup$


                                                                                                                                                                                        Wolfram Language (Mathematica), 26 bytes



                                                                                                                                                                                        If[#<3,1,#0[#-2]+#0[#-3]]&


                                                                                                                                                                                        Try it online!






                                                                                                                                                                                        share|improve this answer









                                                                                                                                                                                        $endgroup$


















                                                                                                                                                                                          1












                                                                                                                                                                                          $begingroup$


                                                                                                                                                                                          Wolfram Language (Mathematica), 26 bytes



                                                                                                                                                                                          If[#<3,1,#0[#-2]+#0[#-3]]&


                                                                                                                                                                                          Try it online!






                                                                                                                                                                                          share|improve this answer









                                                                                                                                                                                          $endgroup$
















                                                                                                                                                                                            1












                                                                                                                                                                                            1








                                                                                                                                                                                            1





                                                                                                                                                                                            $begingroup$


                                                                                                                                                                                            Wolfram Language (Mathematica), 26 bytes



                                                                                                                                                                                            If[#<3,1,#0[#-2]+#0[#-3]]&


                                                                                                                                                                                            Try it online!






                                                                                                                                                                                            share|improve this answer









                                                                                                                                                                                            $endgroup$




                                                                                                                                                                                            Wolfram Language (Mathematica), 26 bytes



                                                                                                                                                                                            If[#<3,1,#0[#-2]+#0[#-3]]&


                                                                                                                                                                                            Try it online!







                                                                                                                                                                                            share|improve this answer












                                                                                                                                                                                            share|improve this answer



                                                                                                                                                                                            share|improve this answer










                                                                                                                                                                                            answered 48 mins ago









                                                                                                                                                                                            attinatattinat

                                                                                                                                                                                            4897




                                                                                                                                                                                            4897






















                                                                                                                                                                                                1 2
                                                                                                                                                                                                next

















                                                                                                                                                                                                draft saved

                                                                                                                                                                                                draft discarded




















































                                                                                                                                                                                                If this is an answer to a challenge…




                                                                                                                                                                                                • …Be sure to follow the challenge specification. However, please refrain from exploiting obvious loopholes. Answers abusing any of the standard loopholes are considered invalid. If you think a specification is unclear or underspecified, comment on the question instead.


                                                                                                                                                                                                • …Try to optimize your score. For instance, answers to code-golf challenges should attempt to be as short as possible. You can always include a readable version of the code in addition to the competitive one.
                                                                                                                                                                                                  Explanations of your answer make it more interesting to read and are very much encouraged.


                                                                                                                                                                                                • …Include a short header which indicates the language(s) of your code and its score, as defined by the challenge.



                                                                                                                                                                                                More generally…




                                                                                                                                                                                                • …Please make sure to answer the question and provide sufficient detail.


                                                                                                                                                                                                • …Avoid asking for help, clarification or responding to other answers (use comments instead).





                                                                                                                                                                                                draft saved


                                                                                                                                                                                                draft discarded














                                                                                                                                                                                                StackExchange.ready(
                                                                                                                                                                                                function () {
                                                                                                                                                                                                StackExchange.openid.initPostLogin('.new-post-login', 'https%3a%2f%2fcodegolf.stackexchange.com%2fquestions%2f182797%2fpatience-young-padovan%23new-answer', 'question_page');
                                                                                                                                                                                                }
                                                                                                                                                                                                );

                                                                                                                                                                                                Post as a guest















                                                                                                                                                                                                Required, but never shown





















































                                                                                                                                                                                                Required, but never shown














                                                                                                                                                                                                Required, but never shown












                                                                                                                                                                                                Required, but never shown







                                                                                                                                                                                                Required, but never shown

































                                                                                                                                                                                                Required, but never shown














                                                                                                                                                                                                Required, but never shown












                                                                                                                                                                                                Required, but never shown







                                                                                                                                                                                                Required, but never shown







                                                                                                                                                                                                Popular posts from this blog

                                                                                                                                                                                                Identifying “long and narrow” polygons in with PostGISlength and width of polygonWhy postgis st_overlaps reports Qgis' “avoid intersections” generated polygon as overlapping with others?Adjusting polygons to boundary and filling holesDrawing polygons with fixed area?How to remove spikes in Polygons with PostGISDeleting sliver polygons after difference operation in QGIS?Snapping boundaries in PostGISSplit polygon into parts adding attributes based on underlying polygon in QGISSplitting overlap between polygons and assign to nearest polygon using PostGIS?Expanding polygons and clipping at midpoint?Removing Intersection of Buffers in Same Layers

                                                                                                                                                                                                Masuk log Menu navigasi

                                                                                                                                                                                                อาณาจักร (ชีววิทยา) ดูเพิ่ม อ้างอิง รายการเลือกการนำทาง10.1086/39456810.5962/bhl.title.447410.1126/science.163.3863.150576276010.1007/BF01796092408502"Phylogenetic structure of the prokaryotic domain: the primary kingdoms"10.1073/pnas.74.11.5088432104270744"Towards a natural system of organisms: proposal for the domains Archaea, Bacteria, and Eucarya"1990PNAS...87.4576W10.1073/pnas.87.12.4576541592112744PubMedJump the queueexpand by handPubMedJump the queueexpand by handPubMedJump the queueexpand by hand"A revised six-kingdom system of life"10.1111/j.1469-185X.1998.tb00030.x9809012"Only six kingdoms of life"10.1098/rspb.2004.2705169172415306349"Kingdoms Protozoa and Chromista and the eozoan root of the eukaryotic tree"10.1098/rsbl.2009.0948288006020031978เพิ่มข้อมูล